You are on page 1of 88

MANAGEMENT ADVISORY SERVICES TRINIDAD/ALENTON/URO

FINAL PRE-BOARD EXAMINATION April 24, 2022

Multiple Choice. Select the letter that corresponds to the best answer. This examination consists of
70 items only. (Please ignore the extra answer options in the answer sheet after number 70.) The exam
is good for three (3) hours. Good luck!

1. Statement I - Presents summary historical data in compliance with generally accepted accounting
principles.
Statement II - Uses cost-benefit analysis to determine the amount of detail presented.
a. Only statement I refers to management accounting
b. Only statement II refers to management accounting
c. Both statements refer to management accounting
d. Neither of the statements refers to management accounting

2. Position I - Chief financial officer (CFO).


Position II - Flight attendant
a. Only position I refers to line management
b. Only position II refers to line management
c. Both positions refer to line management
d. Neither of the positions refers to line management

3. Position I - In-house attorney


Position II - HR manager
a. Only position I refers to line management
b. Only position II refers to line management
c. Both positions refer to line management
d. Neither of the positions refers to line management

4. Advantages of the high-low method include all of the following except


a. only two observations are required to develop the cost function
b. any two observers will arrive at the same conclusion
c. the method is quick to use and easy to understand
d. the data points used represent a typical cost-activity relationships

5. When graphed, a typical variable cost appears as:


a. a horizontal line.
b. a vertical line.
c. a diagonal line that slopes downward to the right.
d. a diagonal line that slopes upward to the right.

6. Dwarf Company currently leases a delivery truck from David Enterprises for a fee of P25,000 per month
plus P40 per mile. Management is evaluating the desirability of switching to a modern, fuel-efficient truck,
which can be leased from Goliath, Inc., for a fee of P60,000 per month plus P5 per mile. All operating costs
and fuel are included in the rental fees. In general, a lease from
a. Goliath, Inc., is economically preferable to a lease from David Enterprises regardless of the monthly use.
b. David Enterprises is economically preferable below 1,000 miles per month.
c. David Enterprises is economically preferable to a lease from Goliath, Inc., regardless of the monthly
use.
d. David Enterprises is economically preferable above 1,000 miles per month.

7. Galaxy Company is preparing a flexible budget for the coming year and the following maximum capacity
estimates for Department 05 are available:
Direct labor hours 60,000
Variable factory overhead P150,000
Fixed factory overhead P240,000

Page 1 of 11
Assume that Galaxy’s normal capacity is 80% of maximum capacity. What would be the total factory
overhead rate, based on direct labors, in a flexible budget at normal capacity?
a. P6.00 c. P7.50
b. P6.50 d. P8.13

8. A company has the following budget formula for annual electricity expense in its shop:
Expense = P7,200 + (Units produced × P0.60)
If management expects to produce 15,000 units during February, the appropriate monthly flexible budget
allowance for the purpose of performance evaluation should be
a. P7,200 c. P9,600
b. P9,000 d. P16,200

9. As an accountant, the most useful information you can get from break-even chart is the
A. relationship among revenues, variable costs, and fixed costs at various levels of activity.
B. volume or output level at which the enterprise breaks even.
C. amount of sales revenue needed to cover enterprise fixed costs.
D. amount of sales revenue needed to cover enterprise variable costs.

10. On a break-even chart, the break-even point is located at the point where the total
A. revenue line crosses the total fixed cost line.
B. revenue line crosses the total contribution margin line.
C. fixed cost line intersects the total variable cost line.
D. revenue line crosses the total cost line.

11. Pare Corporation is a wholesaler that sells a single product. Management has provided the following cost
data for two levels of monthly sales volume. The company sells the product for P133.60 per unit.
Sales volume (units) 4,000 5,000
Cost of sales P383,600 P479,500
Selling, general, and administrative costs P124,400 P136,000
The best estimate of the total contribution margin when 4,300 units are sold is:
a. P112,230 c. P28,380
b. P162,110 d. P45,150

12. The Proway Corporation produces a variety of cleaning compounds and solutions for both industrial and
household use. While most of its products are processed independently, a few are related. “Sprit 357” is a
coarse cleaning powder with many industrial uses. It costs P16 a pound to make and has a selling price
of P20 a pound. A small portion of the annual production of this product is retained for further processing
in the Mixing Department where it is combined with several other ingredients to form a paste which is
marketed as a silver polish selling for P40 a jar. This further processing requires ¼ pound of Sprit 357 per
jar. Other ingredients, labor, and variable overhead associated with this further processing cost P25 per
jar. Variable selling costs amount to P3.00 per jar. If the decision were made to cease production of the
silver polish, P56,000 of fixed Mixing Department costs could be avoided. Assume that the demand for
Sprit 357 is unlimited. The minimum number of jars of silver polish that would have to be sold to justify
further processing of Sprit 357 is
a. 5,600 jars c. 7,000 jars
b. 4,667 jars d. 8,000 jars

13. Ultra Vogue Co. sells 50,000 units of “yo” a top-of-the-line garden sprinkler. These were taken from the
company’s records:
Accounts receivable, P129,000. Contribution margin ratio, 49%.
Days sales outstanding, 15 Profit for the period was
days. P485,040.
The ending receivables balance is the average balance during the year. Assume a 360-day year. All sales
are on credit. Determine the company’s break-even revenue.
a. P2,106,122 c. P1,032,000
b. P3,096,000 d. P1,517,040

Page 2 of 11
14. A summary of expected costs for a range of activity levels that is geared to changes in the level of
productive output is the definition of a
a. continuous budget. d. master budget.
b. flexible budget. e. period budget.
c. static budget

15. The term "management by exception" is best defined as:


a. choosing exceptional managers
b. controlling actions of subordinates through acceptance of management techniques
c. investigating unfavorable variances
d. devoting management time to investigate significant variances

Use the following information for the next three questions.


ChemKing uses a standard costing system in the manufacture of its single product. The 35,000 units of raw
material in inventory were purchased for P105,000, and two units of raw material are required to produce one
unit of final product. In November, the company produced 12,000 units of product. The standard allowed for
material was P60,000, and there was an unfavorable quantity variance of P2,500.

16. ChemKing's standard price for one unit of material is:


a. P2.00 c. P3.00
b. P2.50 d. P5.00

17. The materials price variance for the units used in November was:
a. P2,500 U c. P12,500 U
b. P11,000 U d. P2,500 F

18. The units of material used to produce November output totaled:


a. 12,000 c. 24,000
b. 12,500 d. 25,000

19. Compute the sales volume variance based on the following data:
Budget Actual
Unit sales 20,000 21,000
Unit sales price P30 P32
Unit variable cost P18 P21
a. P30,000 Favorable
b. P12,000 Favorable
c. P30,000 Unfavorable
d. P12,000 Unfavorable

20. If unit costs remain unchanged and sales volume and sales price per unit both increase from the
preceding period when operating profits were earned, operating profits must
A. increase under the absorption costing method.
B. increase under the variable costing method.
C. decrease under the absorption costing method.
D. decrease under the variable costing method.

21. Calculating income under variable costing does not require knowing
A. unit sales. C. selling price.
B. unit variable manufacturing costs. D. unit production.

22. Bach Inc. makes a single product that sells for P40. The standard variable manufacturing cost is P22 and
the standard fixed manufacturing cost is P8, based on producing 30,000 units. During the year Bach
produced 28,000 units and sold 26,000 units. Actual fixed manufacturing costs were P235,000; actual
variable manufacturing costs were P595,000. Selling and administrative expenses were P95,000. There
were no beginning inventories. Based on variable costing, operating income for the year was (M)
A. P175,000 C. P125,000
Page 3 of 11
B. P115,000 D. P159,000

23. __________ method(s) expense(s) variable marketing costs in the period incurred.
a. Variable costing
b. Absorption costing
c. Throughput costing
d. All of these answers are correct.

24. The Glass Shop, a manufacturer of large windows, is experiencing a bottleneck in its plant. Setup time at
one of its workstations has been identified as the culprit. A manager has proposed a plan to reduce setup
time at a cost of P72,000. The change will result in 8,000 additional windows. The selling price per window
is P18, direct labor costs are P3 per window, and the cost of direct materials is P5 per window. Assume all
units produced can be sold. The change will result in an increase in the throughput contribution of:

a. P104,000
b. P80,000
c. P32,000
d. P8,000

25. Clayton Industries is planning its operations for next year, and Ronnie Clayton, the CEO, wants you to
forecast the firm's additional funds needed (AFN). Data for use in your forecast are shown below. Based
on the AFN equation, what is the AFN for the coming year? Pesos are in millions.
Last yr's sales = S0 P350
Sales growth rate =g 30%
Last yr's total assets = A*0 P580
Last yr's prof margin = M 5%
Last yr's accounts payable P40
Last yr's notes payable P50
Last yr's accruals P30
Target payout ratio 60%
a. P120.9
b. P139.6
c. P130.9
d. P143.9

26. An enterprise has excess capacity in production-related property, plant, and equipment. If in a given year
these assets are being used to only 80% of capacity and the sales level in that year is 2 million, the full
capacity sales level is
A. 1,600,000 C. 2,500,000
B. 2,000,000 D. 10,000,000

27. Given an actual demand of 61, forecast of 58, and an α of .3, what would the forecast for the next period
be using simple exponential smoothing?
a. 57.1 c. 61.0
b. 58.9 d. 65.5

Use the following information for the next four questions.


Reliance Company budgets sales at P2,000,000 and expects a net income before tax of 10% of sales.
Expenses are estimated as follows:
Selling 15% of sales
Administrative 9% of sales
Finance 1% of sales
Labor is expected to be 40% of total manufacturing cost. Factory overhead is to be applied at 75% of direct labor
cost.

Page 4 of 11
Inventories are to be as follows:
January 1 December 31
Materials P250,000 P300,000
Work in process 200,000 320,000
Finished goods 350,000 400,000
28. Cost of goods sold will be
A. P500,000 C. P1,300,000
B. P700,000 D. P2,000,000

29. Total manufacturing cost will be


A. P741,000 C. P1,029,000
B. P882,000 D. P1,470,000

30. Factory overhead will amount to


A. P441,000 C. P588,000
B. P491,000 D. P1,029,000

31. Material purchases will be


A. P250,000 C. P491,000
B. P300,000 D. P741,000

32. Management of a company is attempting to build a reputation as a world-class manufacturer of quality


products. Which of the following measures would not be used by the firm to measure quality?
A. The percentage of shipments returned by customers because of poor quality.
B. The number of parts shipped per day.
C. The number of defective parts per million.
D. The percentage of products passing quality tests the first time.

Use the following information for the next three questions.


The Post Division of the M.T. Woodhead Company produces basic posts which can be sold to outside customers
or sold to the Lamp Division of the M.T. Woodhead Company. Last Year the Lamp Division bought all of its 25,000
posts from Post at P1.50 each. The following data are available for last year's activities of the Post Division:
Capacity in units 300,000 posts
Selling price per post to outside customers P1.75
Variable costs per post P0.90
Fixed costs, total P150,000
33. Suppose there is ample capacity so that transfers of the posts to the Lamp Division do not cut into sales
to outside customers. What is the lowest transfer price that would not reduce the profits of the Post
Division?
A. P0.90. C. P1.41.
B. P1.35. D. P1.75.

34. Suppose the transfers of posts to the Lamp Division cut into sales to outside customers by 15,000 units.
What is the lowest transfer price that would not reduce the profits of the Post Division?
A. P0.90. C. P1.41.
B. P1.35. D. P1.75.

35. Suppose the transfers of posts to the Lamp Division cut into sales to outside customers by 15,000 units.
Further suppose that an outside supplier is willing to provide the Lamp Division with basic posts at P1.45
each. If the Lamp Division had chosen to buy all of its posts from the outside supplier instead of the Post
Division, the change in net operating income for the company as a whole would have been:
A. P1,250 decrease. C. P1,000 decrease.
B. P10,250 increase. D. P13,750 decrease.

36. Relevant costs for pricing a special order include


A. existing fixed manufacturing overhead
B nonmanufacturing costs that will not change even if the special order is accepted.
Page 5 of 11
C additional setup costs for the special order.
D all of the above costs.

37. Major factor(s) in evaluating a special order is


A. The expected contribution margin on the order.
B The possible effects on sales at regular prices.
C The availability of capacity to produce the additional units.
D All of the above.

38. Soft Cushion Company is highly decentralized. Each division is empowered to make its own sales
decisions. The Assembly Division can purchase stuffing, a key component, from the Production Division or
from external suppliers. The Production Division has been the major supplier of stuffing in recent years.
The Assembly Division has announced that two external suppliers will be used to purchase the stuffing at
P20 per pound for the next year. The Production Division recently increased its unit price to P40. The
manager of the Production Division presented the following information — variable cost P32 and fixed
cost P8 — to top management in order to attempt to force the Assembly Division to purchase the stuffing
internally. The Assembly Division purchases 20,000 pounds of stuffing per month.

What would be the monthly operating advantage (disadvantage) of purchasing the goods internally,
assuming the external supplier increased its price to P50 per pound and the Production Division is able to
utilize the facilities for other operations, resulting in a monthly cash-operating savings of P30 per pound?
a. P1,000,000 c. P(240,000)
b. P360,000 d. P(400,000)

39. Excellent Corp. produces motherboard at a special economic zone in Central Luzon. It is now considering
to shift to new automated equipment instead of its present facility. Management was given the mandate
to shift if its break even point will materially be improved with a minimum of 10% reduction in volume.
Below are the pertinent information:
Existing With Automation
Sales in units 800,000 900,000
Selling price P30
Variable cost per unit P15 P13
Fixed costs P775,000 P892,500
The company should
a. not shift since the break-even volume will not change.
b. not shift since the break-even volume will even increase by 1% with the automation.
c. shift to automation since the 10% reduction in break-even volume could be achieved.
d. shift to automation since the reduction in break-even volume will be more than 10%

40. The following information pertains to the Ace Company’s three products:
D E F
Unit sales per month 450 700 400
Selling price per unit P2.00 P3.75 P2.50
Variable price per unit 1.00 3.00 2.60
Unit contribution margin P1.00 P0.75 P(0.10)
Assume that product F is discontinued and the space is used to produce E. Product E’s production is
increased to 1,100 units per month, but E’s selling price of all units of E is reduced to P3.40. Monthly profits
will
a. decrease by P345 d. increase by P440
b. increase by P200 e. increase by P125.
c. decrease by P45

Page 6 of 11
Use the following information for the next two questions.
Presented below are excerpts from the income statements of Ralos Company for the years ended December 31,
2021 and 2020.
2021 2020 Inc (Dec)
Net sales P2,750,000 P2,500,000 P250,000
Cost of goods sold 1,584,000 1,600,000 ( 16,000)
Gross margin P1,166,000 P 900,000 P234,000

The unit cost decreased by 10% on January 1, 2021.

41. What is increase (decrease) in cost of goods sold due to volume?


a. P 160,000 c. P(160,000)
b. P(176,000) d. P 176,000

42. What is the effect on sales due to change in selling price?


a. P250,000 increase c. P 75,000 increase
b. P275,000 increase d. P 0

43. Which of the following ratios measures long-term solvency?


A. Acid-test ratio C. Debt to total assets
B. Receivables turnover D. Current ratio

44. If the transaction causes total liabilities to decrease, but does not affect the owners’ equity, what change if
any, will occur in total assets?
A. Assets will be increased. C. No change in total assets.
B. Assets will be decreased. D. None of the above.

Use the following information for the next three questions.


The following are the selected account balances or total as of January 1, 2021:
Accounts receivable P380,000
Merchandise inventory P260,000
Current liabilities P280,000
Selected relationships for the year:
Gross profit rate 40 percent
DSO based on average 40 days
Inventory turnover based on COGS 8X
Current ratio 3:1
Acid Test ratio 2:1

45. The balance of inventory, December 31, 2021 was


a. P260,000 c. P280,000
b. P270,000 d. P300,000

46. The cost of goods sold during 2021 amounted to


a. P2,080,000 c. P2,240.000
b. P2,160,000 d. P2,400,000

47. The balance of accounts receivable as of December 31, 2021 is


a. P400,000 c. P380,000
b. P420,000 d. P390,000

48. If everything else remains constant and a firm increases its cash conversion cycle, its profitability will
likely
a. Increase.
b. Increase if earnings are positive.
c. Decrease.
Page 7 of 11
d. Not be affected.

49. When a specified level of safety stock is carried for an item in inventory, the average inventory level for
that item
a. decreased by the amount of the safety stock.
b. is one-half the level of the safety stock.
c. Increases by one-half the amount of the safety stock.
d. Increases by the amount of the safety stock.

50. Emperor Company obtained a short-term bank loan for P1,000,000 at an annual interest rate 10%. As a
condition of the loan Emperor required to maintain a compensating balance of P200,000 in its checking
account. The checking account earns interest at an annual rate of 3%. Emperor would otherwise maintain
only P75,000 in its checking account for transactional purposes. Emperor’s effective interest costs of the
loan is
a. 10.74% c. 11.75%
b. 10.00% d. 11.00%

51. Lion Company has average receivables of P120,000, which turns over every 30 days. It pledges all of its
receivables to a bank that advances 75% of the total collectibles at 2% over prime rate and charges a 1%
administrative fee on the total amount pledged. The administrative fee is payable at the end of 30 days. If
prime rate is 10%, what effective interest rate is Lion paying for its receivables financing?
a. 24.0% c. 15.0%
b. 13.3% d. 28.0%

52. Rublin Company applies for a one-year, P1 million loan from RY Bank. The term of the loan requires 12%
per annum interest, compounded semi-monthly and will be paid together with the principal at the end of
the year. How much total interest will be paid at the end of the year?
a. P137,870 c. P132,796
b. P136,036 d. P127,160

53. Which of the following statements is false?


a. The most commonly used index with which to measure risk is the standard deviation.
b. The standard deviation measures the average deviation of the various outcomes from the expected
rate of return.
c. The variance is the square root of the standard deviation.
d. The greater the volatility, the larger the standard deviation.

54. Supernormal growth refers to a firm that increases its dividend by:
a. three or more percent per year.
b. a rate which is most likely not sustainable over an extended period of time.
c. a constant rate of 2 or more percent per year.
d. P0.10 or more per year.

55. The Beta Corporation asks you to determine its marginal cost of capital. Beta’s current capital structure
consists of 45% debt, 15% preferred stock and 40% common equity. The separate marginal costs of the
various components of the capital structure are as follows: debt, after-tax 5.0 percent; preferred stock, 9
percent; retained earnings, 12 percent; and new common stock, 13.5 percent. If Beta has P15 million
investible retained earnings, and Beta has an opportunity to invest in an attractive project that costs P60
million, what is the marginal cost of capital of Beta Corporation?
A. 8.40 percent C. P9.00 percent
B. 8.63 percent D. P9.88 percent

56. Wiley’s new financing will be in proportion to the market value of its present financing, shown below.
Book Value (P000 Omitted)
Long-term debt P7,000
Preferred stock (100 1,000
shares)
Page 8 of 11
Common stock (200 7,000
shares)
The firms’ bonds are currently selling at 80% of par, generating a current market yield of 9%, and the
corporation has a 40% tax rate. The preferred stock is selling at its par value and pays a 6% dividend. The
common stock has a current market value of P40 and is expected to pay a P1.20 per share dividend this
fiscal year. Dividend growth is expected to be 10% per year. Wiley’s weighted-average cost of capital is
(round your calculations to tenths of a percent)
a. 13.0% c. 9.6%
b. 8.3% d. 9.0%

57. You are considering an investment in the common stock of Cowher Corp. The stock is expected to pay a
dividend of P2 per share at the end of the year (i.e., D1 = P2.0 ). The stock has a beta equal to 1.2. The risk-
free rate is 6 percent. The market risk premium is 5 percent. The stock's dividend is expected to grow at
some constant rate, g. The stock currently sells for P40 a share. Assuming the market is in equilibrium,
what does the market believe the stock price will be at the end of three years? (In other words, what is P3?)
a. P40.00 c. P42.35
b. P45.67 d. P49.00

58. A project that has a positive NPV discounted at a rate of 15% would have an IRR of
a. 0% why zero? c. More than 15%
b. 10% d. Less than 15%

59. CBN Products, Inc. is considering to invest in one of two projects. Both projects have a net present value
of P25,000; however Project #1 requires an initial investment of P700,000 while Project #2 requires an
initial investment of P300,000. Based on this information, which of the following statements is true?
a. Project #2 will have a higher profitability index
b. Project #1 will have a higher profitability index
c. Both project will have the same profitability index
d. There is not enough information to determine the profitability index of either project

60. The financial management team of a company is assessing an investment proposal involving a P100,000
outlay today. Manager number one expects the project to provide cash inflows of P20,000 at the end of
each year for six years. She considers the project to be of low risk, requiring only a 10% rate of return.
Manager number two expects the project to provide cash inflows of P5,000 at the end of the first year,
followed by P23,000 at the end of each year in years two through six. He considers the project to be of
medium risk, requiring a 14%rate of return. Manager number three expects the project to be of high risk,
providing one large cash inflow of P135,000 at the end of the sixth year. She proposes a 15% rate of return
for the project. According to the net present value criterion, which of the following is true?
a. Manager one will recommend that the project be accepted.
b. Manger two will recommend that the project be accepted.
c. All three managers will recommend acceptance of the project.
d. All three managers will recommend rejection of the project.

Use the following information for the next two questions.


The Duval Corporation has recently evaluated a proposal to invest in cost-reducing production technology.
According to the evaluation, the project would require an initial investment of P17,166 and would provide equal
annual cost savings for five years. Based on a 10 percent discount rate, the project generates a net present value
of P1,788. The project is not expected to have any salvage value at the end of its five-year life.
61. What is the expected annual cost savings of the project?
a. P3,500 c. P4,500
b. P4,000 d. P5,000

62. What is the project's expected internal rate of return?


a. 10% c. 13%
b. 11% d. 14%

63. A basic assumption of activity-based costing (ABC) is that


Page 9 of 11
A. all manufacturing costs vary directly with units of production.
B. products or services require the performance of activities, and activities consume resources.
C. only costs that respond to unit-level drivers are product costs.
D. only variable costs are included in activity-cost pools.

64. Arranging for a shipment of a number of different products to a customer is an example of an activity at
which of the following levels?
A. Unit-level activity. C. Customer-level activity.
B. Batch-level activity. D. Organization-sustaining activity.

65. Jones Construction currently uses traditional costing where overhead is applied based on direct labor
hours. Using traditional costing, the applied overhead rate is P20 per direct labor hour.

They are considering a switch to activity-based costing (ABC). The company controller has come up with
preliminary overhead rates for each of the following activities:
Activity Allocation Base Overhead rate
Material delivery and handling Number of deliveries P100 per delivery
Inspections Number of inspections P75 per inspection
Supervision Hours of supervisor time P30 per supervisor hour
Purchasing Number of purchase P60 per purchase order
orders

One of the company's current jobs has the following information available:

Direct labor hours 50 hours


Number of deliveries 2
Number of inspections 3
Hours of supervisor time 2
Number of purchase orders 5

Which of the following statements is true when comparing the total overhead allocated to the job using
traditional versus ABC costing?
a. ABC costing will yield P215 less in overhead cost being allocated to the job.
b. ABC costing will yield P735 less in overhead cost being allocated to the job.
c. ABC costing will yield P545 more in overhead cost being allocated to the job.
d. ABC costing will yield P785 more in overhead cost being allocated to the job.

Use the following information for the next two questions.


Number of workers Total units of product Average selling price
10 20 P50
11 25 P49
12 28 P47.5
66. The marginal physical product when one worker is added to a team of 10 workers is
a. 1 unit
b. 8 units
c. 5 units
d. 25 units

67. The marginal revenue per unit when one worker is added to a team of 11 workers is
a. P105
b. P225
c. P35
d. P47.5

Page 10 of 11
68. Given the following data, what is the marginal propensity to consume?
Level of
Disposable income Consumption
P40,000 P38,000
48,000 44,000
a. 1.33 c. 0.95
b. 1.16 d. 0.75

69. When several unit sales volumes are multiplied by the probability of their occurrence and those products
are summed, the result is the:
A. median D. best estimated sales level
B. standard deviation E. average sales level
C. expected value

70. A management consultant is scheduling a long-term research and development project. The time table is
very tight due to the advent of the rainy season and to inadequate research and development staff
complement. As the requirements are extensive and complex, what is the most appropriate approach for
planning and controlling the government highway project?
a. Time-series or trend regression analysis.
b. Cost-volume-profit analysis.
c. Queuing theory analysis.
d. Program evaluation review technique

End of Examination

(Pleases ignore the extra answer options in the answer sheet after number 70)

Thank you for participating in


Team PRTC Nationwide Online Open Final Pre-Board Examination for May 2022 LECPA!

Page 11 of 11
AUDITING OCAMPO/OCAMPO/SOLIMAN/UY/RICAFRENTE
FINAL PRE-BOARD EXAMINATION April 24, 2022

Multiple Choice. Select the letter that corresponds to the best answer. This examination consists of
70 items only. (Please ignore the extra answer options in the answer sheet after number 70.) The exam
is good for three (3) hours. Good luck!

1. Sustainability reporting can incorporate which of the following?


a. environmental reporting
b. social reporting
c. business viability reporting
d. all of the above

2. An operational audit has as one of its objectives to:


a. determine whether the financial statements fairly present the entity’s operations.
b. evaluate the feasibility of attaining the entity’s operational objectives.
c. make recommendations for improving performance.
d. report on the entity’s relative success in attaining profit maximization.

3. Prior to beginning the fieldwork on a new audit engagement in which he does not possess industry
expertise, the CPA should
a. reduce audit risk by lowering the preliminary levels of materiality.
b. design special substantive tests to compensate for the lack of industry expertise.
c. engage financial experts who are familiar with the nature of the industry.
d. obtain a knowledge of matters that relates to the nature of the entity's business and industry.

4. In which of the following order would the auditors perform the following steps?
a. Determine audit risk; assess control risk; determine detection risk; set materiality.
b. Set materiality; determine audit risk; assess control risk; determine detection risk.
c. Set materiality; assess control risk; determine detection risk; determine audit risk.
d. Determine audit risk; set materiality; assess control risk; determine detection risk.

5. An auditor need not abide by a particular auditing standard if the auditor believes that:
a. the issue in question is immaterial in amount.
b. more expertise is needed to fulfill the requirement.
c. the requirement of the standard has not been addressed by the AASC.
d. any of the above three are correct.

6. In connection with the examination of financial statements by an independent auditor, the client suggests
that members of the internal audit staff be utilized to minimize audit costs. Which of the following tasks
could most appropriately be delegated to the internal audit staff?
a. Selection of accounts receivable for confirmation, based upon the internal auditor's judgment as to how
many accounts and which accounts will provide sufficient coverage.
b. Preparation of schedules for negative accounts receivable responses.
c. Evaluation of the internal control for accounts receivable and sales.
d. Determination of the adequacy of the allowance for doubtful accounts.

7. An auditor evaluates the existing internal control in order to


a. Determine the extent of substantive tests which must be performed.
b. Determine the extent of control tests which must be performed.
c. Ascertain whether irregularities are probable.
d. Ascertain whether any employees have incompatible functions.

8. During the review of a small business client's internal control system, the auditor discovered that the
accounts receivable clerk approves credit memos and has access to cash. Which of the following controls
would be most effective in offsetting this weakness?
Page 1 of 14
a. The owner reviews errors in billings to customers and postings to the subsidiary ledger.
b. The controller receives the monthly bank statement directly and reconciles the checking accounts.
c. The owner reviews credit memos after they are recorded.
d. The controller reconciles the total of the detail accounts receivable accounts to the amount shown in
the ledger.

9. “Absence of reasonable care that can be expected of a person in a set of circumstances” defines:
a. pecuniary negligence.
b. gross negligence.
c. extreme negligence.
d. ordinary negligence.

10. Of the following audit procedures, which best supports the valuation objective?
a. Performing a lower of cost or market test of the client's inventories.
b. Reviewing a contingent liability footnote for proper wording.
c. Searching for unrecorded liabilities.
d. Observing the client's year-end physical inventory taking.

11. Which of the following statements is correct with respect to the auditor’s use of analytical procedures?
a. Analytical procedures are time saving procedures that auditors may employ at their discretion.
b. Analytical procedures are powerful tools that are required to be used during the planning and testing
phases of the audit.
c. Analytical procedures may be used to identify misstatements in a client’s accounts.
d. Analytical procedures are required to be used during the planning and completion phases of the audit.

12. The inventory of a subsidiary consists of 12,980 items valued at P19,625,000. The inventory data are not
available in computer readable form, but are contained in 217 pages of printout with 60 lines per page.
Each page has a page total. Based on the last audit, the auditor expects there to be a few errors of
overstatement. The most efficient sampling technique to test the reasonableness of the stated value
is
a. Stratified mean-per-unit estimation.
b. Probability proportional to size sampling (PPS)
c. Difference estimation.
d. Ratio estimation.

13. A system of internal control, regardless of how carefully designed and implemented, contains certain
inherent limitations. Which of the following errors or irregularities is not caused by an inherent limitation.
a. The president and chief executive officer, with the assistance of the corporate controller, inflated
earnings by recording fictitious sales at year-end.
b. A newly-installed electronic data processing system failed to provide for a comparison of sales order
amount with prior customer balance and credit limit. This resulted in numerous sales to customers
who had already exceeded their credit limits.
c. Numerous recording errors occurred because persons analyzing and recording transactions did not
have the necessary accounting background.
d. A computer programmer and a computer operator conspired to divert funds from the company to an
account controlled by the dishonest employees.

14. The most effective control for ensuring that customers are billed only for goods shipped is to
a. Require that carriers sign properly completed bills of lading.
b. Implement a policy that prevents the mailing of sales invoices to customers in the absence of a
properly approved shipping order and a bill of lading signed by the carrier.
c. Require that all shipments be approved by accounting.
d. Prohibit goods from leaving the warehouse without being accompanied by a signed bill of lading and a
properly approved shipping order.

Page 2 of 14
15. The possibility of losing a large amount of information stored in computer files most likely would be
reduced by the use of
a. Back-up files
b. Check digits
c. Completeness tests
d. Conversion verification.

16. The auditor issued a qualified opinion covering the financial statements of Client A for the year ended
December 31, 2021. The reason for the qualification was a departure from GAAP. In presenting
comparative statements for the years ended December 31, 2021 and 2022, the client revised the 2021
financial statements to correct the previous departure from GAAP. The auditor's 2022 report on the
12/31/21 and 12/31/22 comparative financial statements will
a. Express a qualified opinion on the 2021 financial statements and an unqualified opinion on the 2022
statements.
b. Express unqualified opinions on both the 2021 and 2022 financial statements.
c. Retain the qualified opinion covering the 2021 statements, but add an explanatory paragraph describing
the correction of the prior departure from GAAP.
d. Render qualified audit opinions for both 2021 and 2022 financial statements given the 2022 carryover
effect of the 2021 error.

17. An auditor is verifying the existence of newly acquired fixed assets recorded in the accounting records.
Which of the following is the best evidence to help achieve this objective?
a. Documentary support obtained by vouching entries to subsidiary records and invoices.
b. Physical examination of a sample of newly recorded fixed assets.
c. Oral evidence obtained by discussions with operating management.
d. Documentary support obtained by reviewing titles and tax returns.

18. Which of the following best describes the auditor's responsibility for "other information" included in the
annual report to stockholders which contains financial statements and the auditor's report?
a. The auditor has no obligation to read the "other information."
b. The auditor has no obligation to corroborate the "other information," but should read the "other
information" to determine whether it is materially inconsistent with the financial statements.
c. The auditor should extend the examination to the extent necessary to verify the "other information."
d. The auditor must modify the auditor's report to state that the "other information is unaudited" or "not
covered by the auditor's report."

19. The auditor should normally concentrate on the key factors and assumptions used by management
including all of the following except those that are
a. insignificant to the accounting estimates.
b. sensitive to variations.
c. deviations from historical patterns.
d. susceptible to misstatements and biases.

20. Which of the following appropriately describes the auditor’s procedures with respect to subsequent
events?
a. The procedures to identity events that may require adjustments of, or disclosure in, the financial
statements would be performed as early as practicable.
b. Those routine procedures that are applied to specific transactions occurring after the period ends are
designed to obtain sufficient appropriate audit evidence that all events up to the date of the audit report
have been identified.
c. When a component is audited by another CPA, the auditor would consider the other auditor’s procedures
regarding events after period end and the need to inform the other auditor of the planned date of the
audit report.
d. The auditor is responsible to inquire regarding the financial statements after the date of the auditor’s
report.

21. Which of the following procedures is not included in a review of financial statements of a nonpublic entity?
Page 3 of 14
a. Inquiries of management
b. Inquiries regarding events subsequent to the balance sheet date
c. Any procedures designed to identify relationships among data that appear to be unusual
d. Communicating any material weaknesses discovered during the study and evaluation of internal
accounting control

22. In determining the sufficiency of evidential matter, which of the following would not normally be a factor?
a. Cost/benefit considerations
b. The sampling technique used
c. Audit risk
d. Materiality of the account

23. The professional accountants issued the following statement in their report: …, nothing came to our
attention that caused us to believe that the accompanying financial statements are not presented fairly, …
What is the nature of the report?
a. Special report on financial statements prepared under comprehensive basis of accounting
b. Qualified audit report
c. Review report
d. Audit report with limited reporting objective

24. Audit working papers are indexed by means of reference numbers. The primary purpose of indexing is to
a. permit cross-referencing and simplify supervisory review.
b. support the audit report.
c. eliminate the need for follow-up reviews.
d. determine that working papers adequately support the findings, conclusions, and reports.

25. Which one of the following is not considered a valid source of information about the client's processes?
a. Confirmation from third-parties
b. Review of the client's budget
c. A tour of the client's plant facility
d. Management inquiry

26. Which of the following statements is incorrect about audit evidence?


a. Evidence obtained from an independent source outside the client organization is more reliable than that
obtained from within.
b. Documentary evidence is more reliable when it is received by the auditor directly from an independent
third party.
c. Documents that originate outside the company are considered more reliable than those that originate
within the client’s organization.
d. External evidence, such as communications from banks, is generally regarded as more reliable than the
information obtained from the client.

27. Which of the following is not likely a source of information about the accounting system in the revenue
area?
a. Direct inquiry of customers.
b. Prior experience with the client.
c. Systems flowcharts prepared by the EDP department.
d. Financial reporting manuals.

28. What sequence of steps does an auditor undertake when identifying control procedures that are potentially
reliable in assessing control risk below the maximum?
a. Consider the errors or frauds that might occur, determine control procedures, identify control objectives,
and design tests of controls.
b. Determine control procedures, design tests of controls, consider the errors or frauds that might occur,
and identify control objectives.
c. Identify control objectives, consider the errors or frauds that might occur, determine control procedures,
and design tests of controls.
Page 4 of 14
d. Design tests of controls, determine control procedures, consider the errors or frauds that might occur,
and identify control objectives.

29. To achieve control when there is no billing department, the billing function should be performed by the
a. accounting department.
b. sales department.
c. shipping department.
d. credit and collection department.

30. In considering internal control within the revenue/receipt cycle, what is the purpose of a transaction walk
through?
a. To gain an assurance that employees are performing assigned functions accurately.
b. To confirm the results of the auditor’s understanding of the internal control structure.
c. To select documents for detailed tests of controls.
d. To verify the results of the auditor’s sampling plan.

31. A company has a policy of rotating employees’ assigned duties. This policy is most important for
employees who:
a. are not bonded.
b. maintain the detailed accounting records.
c. handle cash receipts.
d. have access to the general ledger.

32. While performing an audit of cash, an auditor begins to suspect kiting. Which of the following is the best
evidence that the auditor could obtain concerning whether kiting is taking place?
a. Documentary evidence obtained by vouching entries in the cash account to supporting documents.
b. Documentary evidence obtained by vouching credits on the latest bank statement to supporting
documents.
c. Evidence obtained by preparing a schedule of interbank transfers.
d. Oral evidence obtained through discussions with controller personnel.

33. Which of the following best explains why accounts payable confirmation procedures are not always used?
a. Inclusion of representations on accounts payable in the client representation letter eliminates the need
in most situations.
b. Accounts payable generally are immaterial and may be audited through using analytical procedures.
c. Creditors will press for payment when they receive the confirmation.
d. Confirmations are better at identifying overstatements than understatements, and overstatements are
not typically the major concern with accounts payable.

34. Client's inventory instructions should include all the following except
a. names of client personnel responsible for the count.
b. instructions for recording accurate descriptions.
c. instructions for auditors' test counts.
d. plans for controlling movement of goods.

35. The body mandated by law to promulgate rules and regulations affecting the practice of Accountancy.
a. Professional Regulation commission
b. Philippine Institute of Certified Public Accountants
c. Professional Regulatory Board of Accountancy
d. Commission on Higher Education

Page 5 of 14
PROBLEM NO. 1

Tae-mu Company engaged you to examine its books and records for the fiscal year ended June 30, 2022. The
company’s accountant has furnished you not only the copy of trial balance as of June 30, 2022 but also the copy
of company’s statement of financial position and statement of comprehensive income as at said date. The
following data appears in the cost of goods sold section of the statement of comprehensive income:

Inventory, July 1, 2021 P 500,000


Add Purchases 3,600,000
Total goods available for sale 4,100,000
Less Inventory, June 30, 2022 700,000
Cost of goods sold P3,400,000

The beginning and ending inventories of the year were ascertained thru physical count except that no reconciling
items were considered. Even though the books have been closed, your working paper trial balance show all
account with activity during the year. All purchases are FOB shipping point. The company is on a periodic
inventory basis.

In your examination of inventory cut-offs at the beginning and end of the year, you took note of the following:

July 1, 2021

a. June invoices totaling to P130,000 were entered in the voucher register in June. The corresponding goods
not received until July.

b. Invoices totaling P54,000 were entered in the voucher register in July but the goods received during June.

June 30, 2022

c. Invoices with an aggregate value of P186,000 were entered in the voucher register in July, and the goods
were received in July. The invoices, however, were date June.

d. June invoices totaling P74,000 were entered in the voucher register in June but the goods were not received
until July.

e. Invoices totaling P108,000 (the corresponding goods for which were received in June) were entered the
voucher register, July.

f. Sales on account in the total amount of P176,000 were made on June 30 and the goods delivered at that
time. Book entries relating to the sales were made in June.

QUESTIONS:

Based on the given information and the result of your cut-off tests, answer the following:

36. How much is the adjusted Inventory as of July 1, 2021?


a. P370,000 c. P576,000
b. P500,000 d. P630,000

37. How much is the adjusted Purchases for the fiscal year ended June 30, 2022?
a. P3,600,000 c. P3,894,000
b. P3,840,000 d. P3,914,000

38. How much is the adjusted Inventory as of June 30, 2022?


a. P500,000 c. P892,000
b. P784,000 d. P960,000

Page 6 of 14
39. Which statement is incorrect regarding audit of inventories in accordance with Philippine Standards on
Auditing?
a. When inventory is material to the financial statements, the auditor shall obtain sufficient appropriate
audit evidence regarding the existence and condition of inventory by attendance at physical inventory
counting, unless impracticable.
b. The procedures involve in the attendance at physical may serve as test of controls or substantive
procedures depending on the auditor’s risk assessment, planned approach and the specific procedures
carried out.
c. If attendance at physical inventory counting is impracticable, the auditor shall modify the opinion in the
auditor’s report.
d. When inventory under the custody and control of a third party is material to the financial statements, the
auditor may obtain sufficient appropriate audit evidence regarding the existence and condition of that
inventory by requesting confirmation from the third party as to the quantities and condition of inventory
held on behalf of the entity.

40. Which statement is incorrect regarding audit of inventories?


a. Attendance at physical inventory counting involves inspecting the inventory to ascertain its existence
and evaluate its condition, and performing test counts.
b. Attendance at physical inventory counting is impracticable where inventory is held in a location that may
pose threats to the safety of the auditor.
c. Inspection of documentation of the subsequent sale of specific inventory items purchased prior to the
physical inventory counting may provide sufficient appropriate audit evidence about the existence and
condition of inventory if attendance at physical inventory counting is impracticable.
d. Verifying that all inventory owned by the client is on hand at the time of the count is one of the
independent auditor's objectives in the audit of inventories.

PROBLEM NO. 2

The client, Ha-ri Corp., obtained bank statements for Nov. 30 and Dec. 31, 2022 and reconciled the balances.
You obtained directly the statements of Jan. 12, 2023 and obtained the necessary confirmation. You have found
that there are no errors in addition or subtraction in the client’s books.

11/30/22 12/31/22
Balance, bank statement P344,420 P275,020
Balance, company records 271,260 226,010
Deposits in transit 35,000 ?
Outstanding checks 88,240 ?

12/1-31/22 1/1-12/23

Receipts, cash records P 963,230 P 292,500


Credits, bank statement 941,010 321,490
Disbursements, cash records 1,008,480 177,570
Charges, bank statement 1,010,410 230,180

The following information also was obtained:

a) Check no. 804 for P340 cleared by the bank in Dec. as P1,340. This was found in proving the bank statement.
The bank made the correction on Jan. 8, 2023.

b) A note of P20,000, sent to the bank for collection on Nov. 15, 2022, was collected and credited to the account
on Nov. 28, 2022, net of a collection fee of P80. The note was recorded in the cash receipts on Dec. 21, 2022,
at which date the collection fee was entered as a disbursement.

c) The client records returned checks in red in the cash receipts journal. The checks listed in the table were
returned by the bank.
Page 7 of 14
Amount Returned Recorded Redeposited
Co. A P3,270 12/6/22 No entries 12/8/22
Co. B 6,730 12/27/22 1/3/23 1/15/23

d) Two payroll checks for employee’s vacations totaling P5,500 were drawn on Jan. 3, 2023, and cleared the
bank on Jan. 8, 2023. These checks were not entered in the clients records because semi-monthly payroll
summaries are entered only on the 15th and the last day of each month.

QUESTIONS:

Based on the above and the result of your audit, compute for the following:

41. Deposits in transit as of Dec. 31, 2022


a. P37,220 c. P40,570
b. P40,490 d. P57,220

42. Outstanding checks as of Dec. 31, 2022


a. P86,310 c. P97,230
b. P93,960 d. P97,310

43. Adjusted cash balance as of Dec. 31, 2022


a. P217,280 c. P219,280
b. P218,280 d. P246,930

44. Deposits in transit as of Jan. 12, 2023


a. P12,500 c. P19,310
b. P15,960 d. P19,230

45. Outstanding checks as of Jan. 12, 2023


a. P33,700 c. P50,120
b. P46,850 d. P56,850

PROBLEM NO. 3

On Jan. 1, 2021, CHAIRMAN Company established significant influence over KANG Company by acquiring
60,000 ordinary shares, a 30% interest, for P570,000. The book value of KANG’s equity was P1,300,000 on Jan.
1, 2021. Since this purchase KANG earned profit and paid dividends as follows:

Year Profit Dividends


2021 P180,000 P100,000
2022 310,000 140,000

The fair value per share of KANG’s ordinary shares on Dec. 31, 2021 and 2022 was P8 and P9, respectively.
CHAIRMAN incorrectly accounted for this investment as a financial asset at fair value through profit or loss.

QUESTIONS:

Based on the given information and the result of your audit, answer the following: (Disregard income taxes)

46. The carrying amount of Investment in KANG Company as of Dec. 31, 2021 is understated by
a. P24,000 c. P114,000
b. P90,000 d. P144,000

47. Profit for 2021 is understated by


a. P24,000 c. P114,000
b. P90,000 d. P144,000
Page 8 of 14
48. The carrying amount of Investment in KANG Company as of Dec. 31, 2022 is understated by
a. P51,000 c. P105,000
b. P75,000 d. P165,000

49. Profit for 2022 is misstated by


a. P 9,000 over c. P51,000 under
b. P33,000 under d. P51,000 over

50. Which statement is correct regarding audit of investment in equity instruments?


a. Audited financial statements of the investee company provides the best form of evidence pertaining to
the annual valuation of an investment in which the independent auditor’s client owns a 10% voting
interest.
b. If a client has a large and active investment portfolio that is kept in a bank safe-deposit box and the
auditor is unable to count the securities at the end of the reporting period, the auditor most likely will
request the bank to confirm to the auditor the contents of the safe deposit box at the end of the reporting
period.
c. An audit procedure that provides evidence about proper valuation of equity instruments classified as FA
at FVTPL is confirmation of securities held by broker.
d. To determine whether securities are authentic is not one of the auditor’s primary objectives in an audit
of trading securities.

PROBLEM NO. 4

On Jan. 2, 2021, the Young-seo, Inc. issued P4,000,000 of 8% convertible bonds at par. The bonds will mature on
Jan. 1, 2025 and interest is payable annually every Jan. 1. The bond contract entitles the bondholders to receive
6, P100 par value, ordinary shares in exchange for each P1,000 bond. On the date of issue, the prevailing market
interest rate for similar debt without the conversion option is 10%.
On Dec. 31, 2022, the holders of the bonds with total face value of P2,000,000 exercised their conversion
privilege. In addition, the company reacquired at 110, bonds with a face value of P1,000,000.

QUESTIONS:

Based on the given information and the result of your audit, answer the following:

51. How much of the proceeds from the issuance of convertible bonds should be allocated to equity?
a. Nil c. P 443,328
b. P253,632 d. P1,268,000

52. How much is the carrying amount of the bonds payable as of Dec. 31, 2021?
a. P2,778,800 c. P3,801,005
b. P3,592,340 d. P4,000,000

53. How much is the interest expense for the year 2022?
a. P277,880 c. P359,234
b. P320,000 d. P380,101

54. The conversion of the bonds on Dec. 31, 2022 will increase share premium by
a. Nil c. P730,553
b. P615,786 d. P800,000

55. How much is the loss on bond reacquisition on Dec. 31, 2022?
a. Nil c. P134,724
b. P100,000 d. P192,106

Page 9 of 14
PROBLEM NO. 5

Seong-hun Corp., organized on June 1, 2021, was authorized to issue shares as follows:

• 9% 800,000, P100 par, convertible preference shares


• 2,500,000, P2.50 par value, ordinary shares

During the remainder of the fiscal year ended May 31, 2022, the following transactions were completed in the
order given:

1) 300,000 preference shares were subscribed for at P105, and 900,000 ordinary shares were subscribed for at
P26. Both subscriptions were payable 30% upon subscription, the balance in one payment.

2) The second subscription payment was received, except one subscriber for 60,000 ordinary shares defaulted
on payment. The full amount paid by this subscriber was returned, and all of the fully paid shares was issued.

3) 150,000 ordinary shares were reacquired by purchase at P28.

4) Each preference share was converted into four ordinary shares.

5) The treasury shares were exchanged for machinery with a fair value of P4,300,000.

6) There was a 2-for-1 share split, and the par value of the new ordinary shares is P1.25.

7) Profit was P830,000.

QUESTIONS:

Based on the given information and the result of your audit, determine the following as of May 31, 2022:

56. Share capital - Ordinary shares


a. P2,550,000 c. P2,100,000
b. P5,100,000 d. P4,200,000

57. Total share premium


a. P50,890,000 c. P48,808,000
b. P48,340,000 d. P48,240,000

58. Total contributed capital


a. P53,908,000 c. P55,990,000
b. P53,440,000 d. P53,340,000

59. Total equity


a. P54,270,000 c. P54,738,000
b. P56,820,000 d. P54,170,000

60. When a client company does not maintain its own stock records, the auditor most likely will
a. Inspect the stock book at year-end and accounting for all certificate numbers.
b. Review of the corporate minutes for information as to shares outstanding.
c. Confirm the number of shares outstanding at year-end with the appropriate state official.
d. Obtain written confirmation from the transfer agent and registrar concerning the number of shares issued
and outstanding.

Page 10 of 14
PROBLEM NO. 6

The following trial balance relates to MIN-WOO Corporation at Mar. 31, 2022:

P'000 P'000
Debit Credit
Closing inventories - Mar. 31, 2022 (note (i)) 18,900
Land and building –at valuation (note (iii)) 113,400
Plant and equipment - cost (note (iii)) 64,800
Accumulated depreciation, Apr. 1, 2021 - plant and equipment 30,240
Investment property – valuation Apr. 1, 2021 (note (iii)) 28,800
Trade receivables 38,700
Cash in bank 1,620
Trade payables 21,240
Ordinary shares of P0.25 each 36,000
10% Redeemable preference shares of P1 each 18,000
Revaluation reserve (note (iii)) 37,800
Retained earnings – Apr. 1, 2021 31,500
Profit or loss summary . 88,200
264,600 264,600

The following notes are relevant:

(i) At Mar. 31, 2022, an inventory list based on a physical count had a total cost of P18.9 million. Some
damaged goods that had cost P 1.44 million were included in these. The realizable value of these goods is
expected to be P 1.71 million, provided a remedial work costing P0.81 million is done before they could be
sold.

(ii) Included in the computation of profit or loss are finance costs consisting of interest on overdraft, the full
year's preference dividend and an ordinary dividend of P0.04 per share that was paid in Sept. 2021.

(iii) Non-current assets:

Land and building

A professional valuer submitted a report on Apr. 1, 2021, revaluing the land at P27 million and building at
P86.4 million. The directors decided to incorporate these values in the accounts. On that date the land and
building had a carrying value of P75.6 million and the building had a remaining life of 15 years.

Charge depreciation on a straight-line basis. The entity does not make a transfer to retained earnings in
respect of excess depreciation.

Plant

All plant is depreciated at 12.5% on the reducing balance basis.

Investment property

On Mar. 31, 2022, the investment property was revalued at P24.3 million. The entity uses the fair value
model.

Page 11 of 14
QUESTIONS:

Based on the given information and the result of your audit, answer the following: (Ignore income taxes)

61. The adjusted profit or loss for the year ended Mar. 31, 2022 is
a. P73,080,000 c. P79,380,000
b. P78,840,000 d. P80,640,000

62. The comprehensive income for the year ended Mar. 31, 2022 is
a. P 79,380,000 c. P117,180,000
b. P116,640,000 d. P118,440,000

63. The total assets as of Mar. 31, 2022 is


a. P217,620,000 c. P219,240,000
b. P218,160,000 d. P219,780,000

64. The total liabilities as of Mar. 31, 2022 is


a. P21,240,000 c. P39,240,000
b. P22,860,000 d. P40,860,000

65. The retained earnings balance as of Mar. 31, 2022 is


a. P 98,820,000 c. P105,120,000
b. P104,580,000 d. P107,100,000

PROBLEM NO. 7

Yu-ra Corporation was incorporated on Dec. 1, 2021, and began operations one week later. Before closing the
books for the fiscal year ended Nov. 30, 2022, the controller prepared the following financial statements:

Yu-ra Corporation
Statement of Financial Position
November 30, 2022

Assets
Current assets:
Cash P 150,000
Marketable securities, at cost 60,000
Accounts receivable 450,000
Allowance for doubtful accounts ( 59,000)
Inventories 430,000
Prepaid insurance __15,000
Total current assets 1,046,000
Property, plant and equipment 426,000
Less accumulated depreciation ( 40,000)
Property, plant and equipment, net 386,000
Research and development costs 120,000
Total assets P1,552,000

Liabilities and Shareholders' equity


Current liabilities:
Accounts payable and accrued
expenses P 592,000
Income taxes payable 224,000
Total current liabilities 816,000
Shareholders' equity:
Share capital, P10 par value 400,000
Page 12 of 14
Retained earnings 336,000
Total shareholders' equity 736,000
Total liabilities and shareholders' P1,552,000
equity

Yu-ra Corporation
Statement of Income
For the Fiscal Year Ended November 30, 2022

Net sales P2,950,000

Operating expenses:
Cost of sales 1,670,000
Selling and administrative 650,000
Depreciation 40,000
Research and development 30,000
2,390,000
Income before income taxes 560,000
Provision for income taxes 224 000
Net income P 336,000

Yu-ra is in the process of negotiating a loan for expansion purposes, and the bank has requested audited
financial statements. During the course of the audit, the following additional information was obtained:

a. The investment portfolio consists of short-term investments in marketable equity securities with a total fair
value of P55,000 as of Nov. 30, 2022.

b. Based on an aging of the accounts receivable as of Nov. 30, 2022, it was estimated that P36,000 of the
receivables will be uncollectible.

c. Inventories at Nov. 30, 2022 did not include work in process inventory costing P12,000, sent to an outside
processor on Nov. 29, 2022.

d. A P3,000 insurance premium paid on Nov. 30, 2022 on a policy expiring one year later was charged to
insurance expense.

e. On June 1, 2022, a production machine purchased for P24,000 was charged to repairs and maintenance. Yu-
ra depreciates machines of this type on the straight-line method over a five-year life with no salvage value,
for financial and tax purposes.

f. Research and development costs of P150,000 were incurred the development of a patent, which Yu-ra
expects to be granted during the fiscal year ending Nov. 30, 2023. Yu-ra initiated a five-year amortization of
the P150,000 total cost during the fiscal year ended Nov. 30, 2022.

g. During Dec. 2022, a competitor company filed suit against Yu-ra for patent infringement claiming P200,000
damages. Yu-ra's legal counsel believes that an unfavorable outcome is probable. A reasonable estimate of
the court's award to the plaintiff is P72,500.

h. The 40% effective tax rate was determined to be appropriate for calculating the provision for income taxes
for the fiscal year ended Nov. 30, 2022. Ignore computation of the deferred portion of income taxes.

Page 13 of 14
QUESTIONS:

Based on the given information and the result of your audit, determine the following as of and for the fiscal period
ended Nov. 30, 2022:

66. Net income


a. P235,260 c. P253,260
b. P239,760 d. P283,260

67. Current assets


a. P1,061,000 c. P1,079,000
b. P1,073,000 d. P1,084,000

68. Total assets


a. P1,484,200 c. P1,489,200
b. P1,486,600 d. P1,491,600

69. Total liabilities


a. P783,340 c. P833,340
b. P805,840 d. P855,840

70. Total equity


a. P635,260 c. P653,260
b. P639,760 d. P683,260

End of Examination
(Pleases ignore the extra answer options in the answer sheet after number 70)

Thank you for participating in


Team PRTC Nationwide Online Open Final Pre-Board Examination for May 2022 LECPA!

Page 14 of 14
TAXATION GUDANI/NARANJO/SIAPIAN
FINAL PRE-BOARD EXAMINATION April 25, 2022

Multiple Choice. Select the letter that corresponds to the best answer. This examination consists of
70 items only. (Please ignore the extra answer options in the answer sheet after number 70.) The exam
is good for three (3) hours. Good luck!

Use the tax tables, if necessary:

GRADUATED TAX TABLE UNDER TRAIN LAW (January 1, 2018 to December 31, 2022)

Over But not over The tax shall be Plus Of excess over
250,000 0 0 -
P 250,000 400,000 0 20% P 250,000
400,000 800,000 30,000 25% 400,000
800,000 2,000,000 130,000 30% 800,000
2,000,000 8,000,000 490,000 32% 2,000,000
8,000,000 2,410,000 35% 8,000,000

SCHEDULE OF EXPANDED WITHHOLDING TAX (EWT)

Income Payment EWT Income Payment EWT


Professional/Management/Consultancy Gross receipts 3M Purchase of goods or 1% - goods
fees (Individual) or below - 5% properties by Top 2% - service
Gross receipts over Withholding Agents (TWA)
3M or VAT-reg – -
10%
Professional/Management/Consultancy Gross receipts Partners in general 720,000 or
fees (Non-Individual) 720,000 or below - professional partnerships less -10%
10% (drawings, advances, Above
Gross receipts over sharings, allowances, etc.) 720,000 –
720,000 – 15% 15%
Contractors/subcontractors (security, 2% Commission (if not 10%
janitorial, etc.) employee)
Director’s fees (if not employee) 10% or 15% Income distributed to 15%
(720,000 beneficiaries of
threshold) estates/trusts
Rental 5%

Page 1 of 16
REVISED WITHHOLDING TAX TABLE (COMPENSATION)
Effective January 1, 2018 to December 31, 2022

DAILY 1 2 3 4 5 6
Compensation 685 and 685-1,095 1,096- 2,192-5,478 5,479-21,917 21,918 and
Range below 2,191 above
Prescribed 0 0 82.19 356.16 1,342.47 6,602.74
Withholding Tax + 20% over + 25% over + 30% over + 32% over + 35% over
685 1,096 2,192 5,479 21,918
WEEKLY 1 2 3 4 5 6
Compensation 4,808 and 4,808-7691 7,692- 15,385-38,461 38-462- 153,846 and
Range below 15,384 153,845 above
Prescribed 0 0 576.92 2,500 9,423.08 46,346.15
Withholding Tax + 20% over + 25% over +30% over + 32% over + 35% over
4,808 7,692 15,385 38,462 153,846
SEMI-MONTHLY 1 2 3 4 5 6
Compensation 10,417 10,417- 16,667- 33,333-83,332 83,333- 333,333 and
Range and below 16,666 33,332 333,332 above
Prescribed 0 0 1,250 5,416.67 20,416.67 100,416.67
Withholding Tax + 20% over + 25% over + 30% over + 32% over + 35% over
10,417 16,667 33,333 83,333 333,333
MONTHLY 1 2 3 4 5 6
Compensation 20,833 20,833- 33,333- 66,667- 166,667- 666,667 and
Range and below 33,332 66,666 166,666 666,666 above
Prescribed 0 0 2,500 10,833.33 40,833.33 200,833.33
Withholding Tax + 20% over + 25% over + 30% over + 32% over + 35% over
20,833 33,333 66,667 166,667 666,667

DOCUMENTARY STAMP TAX TABLE

Page 2 of 16
1. S1: Taxes may be imposed to raise revenues or to provide tax incentives to certain activities within the
state. T
S2: The state can have the power of taxation even if the Constitution does not expressly give it the power
to tax. T
S3: For the exercise of the power of taxation, the state can tax anything within the commerce of man at any
time. T
S4: The provisions of taxation in the Philippine Constitution are grants of power and not limitations on
taxing power. F
Which is FALSE?
a. Only S1 is false
b. Only S2 is false
c. Only S3 is false
d. Only S4 is false
e. All is false

2. S1: Real property tax is a national tax.F


S2: Value-added tax is a progressive tax. F
S3: Estate tax is a direct tax. T
a. Only S1 is true
b. Only S2 is true
c. Only S3 is true
d. All is false

3. S1: Non-payment of tax does not necessarily render the business illegal.T
S2: License fee is imposed as a regulatory measure. T
S3: Tax has generally no limit on the amount that may be imposed. T
S4: License fee is generally limited to cover the cost of regulation. T
a. Only S1 is false
b. Only S2 is false
c. Only S3 is false
d. Only S4 is false
e. All is true

4. Facts: In 2020, the government did not meet its target revenue collection because of the Covid-19 pandemic.
In order to sustain its operations, it borrowed money from China.
S1: The borrowing of money from China is consistent with the principle of Administrative Feasibility.Fiscal Adequacy
S2: The borrowing of money from China is in violation of the Constitution as it violates the principle of Fiscal
Adequacy that the government should yield enough revenue to finance its operation.
a. Only S1 is true
b. Only S2 is true
c. Both are true
d. Both are false

5. S1: Direct double taxation is prohibited by the Philippine Constitution. F


S2: Indirect double taxation is not prohibited by the Philippine Constitution. T
a. Only S1 is true
b. Only S2 is true
c. Both are true
d. Both are false

6. S1: The power of taxation involves raising money for the government.T
S2: The power of eminent domain raises money for the government. F
a. Only S1 is true
b. Only S2 is true
c. Both are true
d. Both are false

Page 3 of 16
7. Nicanor, a non-resident citizen residing in United State of America, deposited $5,000 in ABC Bank, a local
bank authorized by the Banko Sentral ng Pilipinas (BSP) to engage in foreign-currency denominated
transactions.
S1: If the deposited money earns $20 interest income, the interest income is subject to 15% final tax. F
S2: If the deposited money earns P1,000 interest income, the interest income is subject to 20% final tax. F
a. Only S1 is true
b. Only S2 is true
c. Both are true
d. Both are false

8. Park, a Filipino-Korean actor, inherited parcel of land located in Clark Pampanga from his mother Inday, a
Filipino residing in Clark Pampanga. Park sold the parcel of land to Nicanor, a Filipino residing in Manila as
evidenced by a deed of sale.
S1: The sale of land by Park is subject to 6% capital gains tax.T
S2: The sale of land by Park is subject to regular income tax. F
S3. The sale is subject to documentary stamp tax. T
a. Only S1 is false
b. Only S2 is false
c. Only S3 is true
d. All is true

9. Nicanor, resident citizen, sold his shares of stock from ABC Corporation, a foreign corporation based in
United Kingdom, to Inday, a Filipino who migrated in Canada, for P 200,000 with a cost of P 100,000.
S1: For income tax purposes, the sale of shares of stock by Nicanor is subject to normal income tax. T
S2: The sale of the shares of stock is subject to 15% capital gains tax based on net capital gain of P 100,000.F
a. Only S1 is true
b. Only S2 is true
c. Both are true
d. Both are false

10. Facts: Nicanor, a Filipino residing in Manila, borrowed P 50,000 from Inday, an OFW (OCW) living in Saudi
Arabia. As a result, Inday earned P 2,000 interest income.
S1: Interest income earned by Inday is an income derived from sources within the Philippines. T
S2: The Interest income earned by Inday is subject to regular or normal tax. T
a. Only S1 is true Situs of Interest Income is the residence of the debtor
b. Only S2 is true
c. Both are true
d. Both are false

11. Vladmir, a Russian citizen residing in the Philippines, borrowed P 100,000 from Xi Jeng, a Chinese residing
in Beijing China. As a result, Xi Jeng earned P 10,000 interest income.
S1: Interest income earned by Xi Jeng is an income derived from sources without the Philippines. F
S2: The Interest income earned by Xi Jeng is taxable.T
a. Only S1 is true Situs of Interest Income is the residence of the debtor
b. Only S2 is true
c. Both are true
d. Both are false

12. Nicanor, a Filipino residing in the United States, earned P 20,000 dividend income from his investment in ABC
Corporation, a corporation established under the laws of the Philippines. The Corporation earned gross
income (for the last three years) in the United States amounting to Php 500M and another Php 200M within
the Philippines.
S1: Dividend income earned by Nicanor is an income derived from sources without the Philippines. F
S2: The Dividend income earned by Nicanor is subject to final tax of 10%. T
a. Only S1 is true
b. Only S2 is true
c. Both are true
Page 4 of 16
d. Both are false

13. Facts: Earnest and Old (EO), is partnership established under the laws of the Philippines. Nicanor, a Filipino
residing in Manila, is working as a Junior auditor of EO. He was assigned by EO to work in Australia for 3
months to audit one of its clients. EO paid Nicanor the amount of P 30,000 per month as his salary.
S1: The salary earned by Nicanor is an income derived from sources within the Philippines. F
S2: The salary earned by Nicanor is taxable. T
a. Only S1 is true
b. Only S2 is true
c. Both are true
d. Both are false

14. Facts: Lisa, a Korean citizen who lives in Seoul Korea, works as an actress in JYP Inc, a corporation
established under the laws of South Korea. One day, Lisa was invited to work with ABC ZBN, a domestic
corporation in the Philippines, for their upcoming movie entitled “My Korean GF” which will be held in the
Philippines. Lisa signed her contract with ABC ZBN which states that she will work with the latter for 3
months and that ABC ZBN will pay JYP Inc P 10,000,000 for the professional fee. JYP Inc pays Lisa the
amount of P 4,000,000 as her professional fee.
S1: The professional fee earned by Lisa is an income derived from sources within the Philippines.T
S2: The professional earned by Lisa is taxable. T
a. Only S1 is true
b. Only S2 is true
c. Both are true
d. Both are false

15. Facts: Rose, a Filipino Citizen residing in Malabon, earns P 200,000 income from renting her house and lot
in Italy.
S1: The rent income earned by Rose is an income derived from sources within the Philippines. F
S2: The rent income earned by Rose is taxable.T
a. Only S1 is true Situs of Rentals is where the property is located
b. Only S2 is true
c. Both are true
d. Both are false

16. Facts: John, an American Citizen residing in the United States of America (USA), applied for a franchise of
Jollibee, a local fastfood chain, in the USA. The Jollibee corporation, a domestic corporation, which is the
owner of the franchise was paid the amount of $ 50,000 in the Philippines representing income from
royalty.
S1: The royalty income earned by Jollibee is an income derived from sources within the Philippines. F
S2: The royalty income earned by Jollibee is subject to final tax. F
a. Only S1 is true Situs of Royalty is the place of use of intangibles
b. Only S2 is true
c. Both are true
d. Both are false

17. S1: In order for passive income to be subject to final tax under section 24 (B) of the NIRC, the income must
be derived from sources within or without the Philippines. F
S2: An income that is exempted from income tax under the law, is not subject to withholding tax. T
S3: Dividend income earned by a resident citizen is always subject to 10% final tax. F
Which is True?
a. Only S1 is True
b. Only S2 is True
c. Only S3 is True
d. All is false

Page 5 of 16
18. Nicanor, a Resident Citizen, bought shares of stocks from ABC Corporation, a corporation established under
the laws of the Philippines. Later, he sold the same for a gain of P 50,000 in the United States of America to
Inday, an American Citizen residing therein. The P 50,000 gain on sale is:
S1: An income derived from sources within the Philippines.
S2: An income of Nicanor subject to capital gains tax.
a. Only S1 is true
b. Only S2 is true
c. Both are true
d. Both are false

19. Piyu Corporation, a domestic corporation, owns twenty (20%) of the outstanding share of NTZ Corporation,
a non-resident foreign corporation (NRFC) since August 15, 2015. On June 30, 2021, it received dividends
amounting to P 1,000,000 from the said NRFC. The said dividend has not been used until January 13, 2023,
to which in the said year, the income is reinvested in the business operation of Piyu Corporation.
S1: The dividend income shall be declared as income for the taxable year 2021.
S2: The dividend income shall be subject to surcharges, interest, and penalty since it was not utilized within
the next table year, which is 2022.
S3: The dividend income is exempted from income tax, as all the conditions for a foreign sourced dividends
received by a domestic corporation to be exempted from income tax under Section 5 of Revenue Regulations
no. 5-2021 are present in this case.
a. Only S1 is false
b. Only S2 is false
c. Only S3 is false
d. All is true

20. S1: Nicanor, a senior manager of ABC Corp., received P 30,000 fringe benefits from the company. Nicanor
has the obligation to report in his annual income tax return the fringe benefit he received.
S2: Inday, an accounting clerk of ABC Corp., received P 5,000 fringe benefits from the company. The fringe
benefits received by Inday is subject to fringe benefits tax.
a. Only S1 is true
b. Only S2 is true
c. Both are true
d. Both are false

21. ABC Corp. grants its employees 20 days vacation leave for the whole year. Such vacation leave may be
converted into cash before the end of the year. Inday, an accounting clerk of the company, who is earning a
rate of P 600 per day, monetized her unused vacation leave of P 6,600 at the end of the year. The whole
amount of P 6,600 monetized unused vacation is:
S1: Part of Inday’s taxable compensation.
S2: Partly treated taxable compensation and partly exempt from income tax.
a. Only S1 is true
b. Only S2 is true
c. Both are true
d. Both are false

22. Department of Finance grants its employees 20 days’ sick leave for the whole year. Such sick leave may be
converted into cash before the end of the year. Nicanor, one of its rank-and-file employees, who is earning a
rate of P 600 per day, monetized his unused sick leave of P 6,600 at the end of the year. The whole amount
of P 6,600 monetized unused vacation is:
S1: Part of Nicanor’s taxable compensation.
S2: Partly treated taxable compensation and partly exempt from income tax.
a. Only S1 is true
b. Only S2 is true
c. Both are true
d. Both are false

Page 6 of 16
23. Which of the following is not a function of Bureau of Customs?
a. Assessment and Collection of all national internal revenue taxes, fees, and charges
b. Simplification and harmonization of customs procedures to facilitate movement of goods in
international trade
c. Border control to prevent entry of smuggled goods
d. Prevention and suppression of smuggling and other customs fraud
e. Facilitation and security of international trade and commerce through an informed compliance program

24. S1 – Capital gains are subject to capital gains tax.


S2 – Ordinary gains are subject to ordinary income tax.
a. Only S1 is true
b. Only S2 is true
c. Both are true
d. Both are false

25. S1 – Capital gains from disposition of capital assets are subject to holding period.
S2 – Ordinary gains from disposition of ordinary assets are NOT subject to holding period.
a. Only S1 is true
b. Only S2 is true
c. Both are true
d. Both are false

26. S1 – Capital assets can be converted to ordinary assets.


S2 – Ordinary assets can be converted to capital assets.
a. Only S1 is true
b. Only S2 is true
c. Both are true
d. Both are false

27. S1 – All dispositions of real properties classified as capital assets are subject to capital gains tax.
S2 – All dispositions of shares of stocks are subject to capital gains tax.
a. Only S1 is true
b. Only S2 is true
c. Both are true
d. Both are false

28. S1 – Sales to senior citizens are deductions from gross income. only the discount granted is deductible
S2 – Sales to persons with disabilities are exempt from VAT.
a. Only S1 is true
b. Only S2 is true
c. Both are true
d. Both are false

29. Nicanor, a Filipino citizen, died in exile in 2019 in the United States of America. He is survived by his wife,
Inday, his son Dayunyor who is the executor named in the will, his daughters Aymi and Ayrin. While
Nicanor’s will is being probated in Court, the BIR collated all his known properties including those which
appeared to be acquired by illegal means. The computed estate tax including amounted to Php 3 Billion.
To date, the estate tax remains unpaid. Who is the person primarily liable to pay the estate tax?
a. Estate of Nicanor
b. Dayunyor only
c. Inday because she is the wife
d. All the heirs

30. Nicanor, a Filipino citizen, died in exile in 1989 in the United States of America. He is survived by his wife,
Inday, his son Dayunyor who is the executor named in the will, his daughters Aymi and Ayrin. While
Nicanor’s will is being probated in Court, the BIR collated all his known properties including those which
Page 7 of 16
appeared to be acquired by illegal means. The computed estate tax including amounted to Php 3 Billion.
To date, the estate tax remains unpaid. Which is TRUE?
S1 – The Commissioner has the power to file the estate tax return of a decedent.
S2 – The estate includes all properties illegally acquired.
a. Only S1 is true
b. Only S2 is true
c. Both are true
d. Both are false

31. Nicanor ran and won the mayoralty position in 2019 midterm elections. During the campaign period, he
received contributions amounting to Php 70M. He was only able to utilize Php 20M. Which is TRUE?
S1 – Nicanor is liable to pay income tax on the unutilized election contributions if he opts not to return the
excess.
S2 – Contributions for election campaign in cash or in kind to any candidate, political party or coalition of
parties are governed by the Election Code.
a. Only S1 is true
b. Only S2 is true
c. Both are true
d. Both are false

32. Nicanor is a CPA. He was engaged by Josh, a resident alien and President of Farmally Corp., a domestic
corporation, to sign and certify its financial statements without audit. Inday, a Revenue Officer, knowing
that Farmally Corp. committed tax evasion, extorted money. Which is TRUE?
S1 – Nicanor’s certificate as a CPA shall be automatically revoked or cancelled upon conviction.
S2 – Josh shall be immediately deported after serving sentence without further proceedings for
deportation.
S3 – Inday committed grave misconduct and shall be subjected to administrative case and subject to Civil
Service Law, shall be dismissed from the revenue service after due notice and hearing.
a. Only S1 is true
b. Only S2 is true
c. Only S3 is true
d. All is true
d. All is false

33. Nicanor is a CPA. He was engaged by Josh, a resident alien and President of Farmally Corp., a domestic
corporation, to sign and certify its financial statements without audit. Inday, a Revenue Officer, knowing
that Farmally Corp. committed tax evasion, extorted money. Which is TRUE? Penalty is:
a. Fine of not less than Php 500,000 and not exceeding Php 10M or imprisonment of not less than 6 years
but not more than 10 years
b. Fine of not less than Php 5M and not exceeding Php 10M and imprisonment of not less than 6 years but
not more than 10 years
c. Fine of not less than Php 500,000 and not exceeding Php 10M and imprisonment of not less than 10
years but not more than 20 years
d. Fine of not less than Php 500,000 and not exceeding Php 10M and imprisonment of not less than 6 years
but not more than 10 years

34. In registration of taxpayers, which is TRUE?


S1 – An employee is required to be registered within 10 days from date of employment.
S2 – A sole proprietor is required to be registered on or before start of business or practice of profession
but may register within 30 days from the issuance of registration from DTI or the issuance of Mayor’s
Permit without incurring penalties for late registration.
S3 – A corporation is generally required to be registered before payment of tax or within 30 days from the
issuance of registration from SEC or of Mayor’s Permit without incurring penalties for late registration.
a. Only S1 is true
b. Only S2 is true
c. Only S3 is true
d. All is true
Page 8 of 16
e. All is false

35. Which is FALSE in registration?


a. Venue of initial registration of employees is the employer’s place of business
b. For self-employed, registration is with BIR which has jurisdiction over the place of business
c. Registration of One-Time Transactions is with RDO where the transaction took place
d. If taxpayer is not an employee nor self-employed, venue is RDO which has jurisdiction over the place of
residence

36. Which is FALSE in payment of Registration Fee with the BIR?


a. Fee is Php 500 whether registration is VAT or non-VAT
b. Php 500 is required to be paid every year after year of registration
c. All taxpayers initially registering with the BIR are required to pay the registration fee
d. Deadline of payment is January 31 of each year following registration

37. Which is FALSE in bookkeeping requirements?


a. All corporations, companies, partnerships or persons required by law to pay internal revenue taxes are
required to keep books of accounts
b. Retention of books of accounts is 10 years
c. Those whose quarterly sales, earnings, receipts, or output do not exceed Php 50,000 shall keep and use
simplified set of bookkeeping records
d. Books of accounts of taxpayers whose gross annual sales exceed Php 3M are required to be examined
by an independent CPA

38. Which is FALSE in deadlines of filing of returns of following transactions?


Transaction Deadline
a. Sale of real property classified as capital asset 30 days from date of notarization of deed
b. Donation of personal property 30 days from date of notarization of deed
c. Estate of decedent Within One year 30 days from date of death of decedent
d. Sale of unlisted shares of stocks 30 days from date of notarization of deed

39. Which is FALSE?


S1 – A Notice of Discrepancy needs to be settled within 30 days from receipt of Notice
S2 – A taxpayer is required to reply to the Preliminary Assessment Notice (PAN) within 15 days from
receipt of the PAN
S3 – A taxpayer is required to protest to the Formal Letter of Demand / Final Assessment Notice
(FLD/FAN) within 30 days from receipt of the FLD/FAN otherwise the assessment shall become final,
executory and demandable
S4 – A taxpayer is required to file an administrative or judicial appeal within 30 days from receipt of the
Final Decision on Disputed Assessment (FDDA) otherwise the assessment shall become final, executory
and demandable.
a. S1
b. S2
c. S3
d. S4

40. Which is TRUE? In the issuance of Formal Letter of Demand (FLD)


S1 – A taxpayer is always required to submit documents in support of its protest. F
S2 – The assessment shall become final, executory and demandable if the taxpayer fails to file a protest. T
a. Only S1 is true
b. Only S2 is true
c. Both are true
d. Both are false

41. Payment of professional tax is with:


a. Provincial Treasurer or Treasurer of Highly Urbanized Cities
b. Municipal Treasurer
Page 9 of 16
c. Office of the Governor or Mayor through City Administrator
d. Sanggunian Panlalawigan or Panlunsod

42. The required document issued by the Bureau of Internal Revenue when processing one-time transactions
like the capital gains tax, estate and donor’s tax and is considered as proof that proper taxes were paid:
a. Certificate Authorizing Registration
b. Certificate of Authority
c. Certificate of Registration
d. Letter of Authority

43. Which is FALSE?


S1 – Remittance Returns of Creditable Income Taxes Withheld (Expanded) are filed every quarter.
S2 – Remittance Returns of Final Income Taxes Withheld are filed every quarter.
S3 – Remittance Returns of Income Taxes Withheld on Compensation are filed every quarter.
a. S1
b. S2
c. S3
d. None is false

44. Which is FALSE?


S1 – VAT Declarations/Returns are filed monthly and quarterly. T
S2 – Percentage Tax Returns of taxpayers whose registration is non-VAT and whose gross receipts are
subject to 1% rate are filed monthly and quarterly. F
a. S1
b. S2
c. Both are true
d. Both are false

45. Which is subject to 12% VAT? Assume all sales are direct costs to the buyers. ABC Corp., a domestic
corporation, sold goods to
S1 – DEF Corp., a PEZA-registered export-oriented enterprise paying Php 1 Million.
S2 – GHI Corp., a corporation registered under the laws of Japan and doing business therein paying Php 1
Million.
S3 – JKL Corp., a local export-oriented enterprise paying $1 Million.
S4 – MNO Corp., a registered business enterprise under CREATE Law, paying $1 Million
a. S1
b. S2
c. S3
d. S4

46. S1 – Nicanor opted 8% taxation. If his gross sales/receipts is Php 5M, he is subject to percentage tax
based on the Php 3M. T
S2 – Nicanor opted 8% taxation. If his gross sales/receipts is Php 5M, he is subject to 12 VAT based on the
excess of Php 2M. T
a. S1
b. S2
c. Both are true
d. Both are false

47. The pertinent facts of the case to the BIR audit for taxable year 2019 are shown below:
Date of receipt of electronic Letter of Authority March 1, 2021 served within 30 days from date of issuance
Date of service and receipt by the taxpayer April 1, 2021
Date of Notice of Discrepancy June 1, 2022
Date of receipt of the PAN July 10, 2022
Date of reply July 31, 2022
Date of receipt of FLD/FAN August 1, 2022
Date of protest September 1, 2022
Page 10 of 16
Based on the above, which is TRUE?
a. The reply is void
b. The assessment is void LOA is void having been served after the lapse of the 30 - day period
c. The assessment is final, executory and demandable
d. The assessment is valid

48. The pertinent facts of the case to the BIR audit for taxable year 2019 are shown below:
Date of receipt of electronic Letter of Authority March 1, 2021 served within 30 days from date of issuance
Date of service and receipt by the taxpayer March 10, 2021
Date of Notice of Discrepancy June 1, 2022 Discussion of discrepancy shall be conducted not later than 30 d
Date of receipt of the PAN July 20, 2022taxpayer shall reply within 15 days from receipt of PAN
Date of reply July 31, 2022
Date of FLD/FAN and date of receipt August 1, 2022 taxpayer shall file a protest within 30 days from receipt of F
Date of protest September 1, 2022

Based on the above, which is TRUE?


a. The assessment is void due to prescription
b. The FLD/FAN is void having been issued within the 15-day period
c. The assessment is final, executory and demandable protest was late having been made after the 30 - day period
d. The assessment is questionable

49. Judicial office which has power to suspend collection of tax:


a. Court of Tax Appeals
b. Supreme Court
c. Court of Appeals
d. Regional Trial Courts

50. In filing a return, which is FALSE?


a. If a taxpayer failed to file returns in previous years, he is not required to file the same if he is in a foreign
country
b. A representative can file the return if authorized
c. The Commissioner can file a return for and on behalf of a taxpayer
d. It is wrong not to file a return if required by law

Below are the data provided to you by Nicanor, a top withholding agent for the month of May 2021:

Purchases of goods from XYZ Corporation, a regular supplier. 1% 800,000


Purchases of services from GHI Corporation, a regular supplier. 2% 200,000
Purchase of services from Jabiiee corporation (meals for employees of 25,000
the company; transacted 4 times)
Purchases of goods from NBS corporation (purchase of office supplies; 8,000
transacted 8 times) 1%
Rent expense 5% 50,000
Professional fees (Individual w/sworn declaration)5% 30,000
Professional fees (Individual w/o sworn declaration) 10% 40,000
Security fees (20% of which is the agency fee) 2% of 20% 100,000
Janitorial services fees 2% 10,000

51. Based on the foregoing data, how much is the total expanded withholding tax required to be withheld?
a. Php 22,280
b. Php 21,780
c. Php 21,180
d. Php 20,680

Page 11 of 16
The following are the data given to you by an employer. You were contracted to handle the payroll:

Employees Name Status Compensation Frequency of payroll


Rey Contractor 537 Daily
(Pakyawan/Arawan)
Ram Probationary 900 Daily
Nicanor Regular 20,000 Weekly
Rian Regular 45,000 Semi-monthly
Bryan Regular 150,000 Monthly

Additional information:
• The company operates 26 working days in the month of September
• Assume there are four (4) weeks in September
• The statutory minimum wage rate is P 537 per day as per regional tripartite wage and productivity
board

52. How much is the total withholding tax on compensation will the employer need to withhold for the month
of September?
a. Php 74,207.27
b. Php 61,406.00
c. Php 48,677.50
d. Php 70,322.77

Nicanor, a Filipino Citizen residing in Manila, had the following data for the taxable year 2021:

Gross income from rent of commercial spaces,


net of P 60,000 withholding tax P 1,140,000

Dividend income:
From domestic corporation Final Tax-10% 50,000
From resident foreign corporation Normal Tax 60,000
From non-resident foreign corporation Normal Tax 20,000

Interest income:
On notes receivable Normal Tax 6,000
On Philippine bank deposit, net of FWT Final Tax-20% 4,800
On Phil bank deposit under FCDU Final Tax-15% 4,000
On bank deposit abroad Normal Tax 5,000
On long-term bank deposit Exempt 20,000

Royalty income:
From literary works Final Tax-10% 10,000
Other than literary works Final Tax-20% 12,000

Information on disposition of capital assets:


Capital gain on sale of unlisted shares of stock of ABC Corp CGT-15% 200,000
On sale of vacation house in Gapan City
• Acquisition Cost - 5,500,000
• Selling price - 6,000,000 CGT-6%
• Zonal Value - 4,000,000
• FMV per tax declaration 5,000,000

Business expenses 691,000

Page 12 of 16
Questions:

53. How much is the taxable income?


a. Php 1,300,000.00
b. Php 650,000.00
c. Php 540,000.00
d. Php 600,000.00

54. How much is the total capital gains tax due?


a. Php 30,000.00
b. Php 330,000.00
c. Php 360,000
d. Php 390,000.00

55. How much is the total passive income subject to final tax (excluding those subjects to capital gains tax)?
a. Php 185,800.00
b. Php 187,000.00
c. Php 80,800.00
d. Php 82,000.00

56. How much is the total final tax due? (Excluding capital gains tax)
a. Php 11,700.00
b. Php 10,500.00
c. Php 9,960.00
d. Php 10,200.00

57. ABC Corporation, a Resident Foreign Corporation, has gross income for the three-year period of P
500,000,000 from sources within the Philippines. It also derived gross income from outside of the
Philippines, amounting to P 300,000,000. In 2022, it declared dividend amounting to Ten Million pesos (P
10,000,000.00). Five Million pesos (P 5,000,000.00) of which was paid to Corporation XYZ, a domestic
corporation.

Based on the foregoing facts, how much is the income tax due of XYZ Corporation?
a. P 1,250,000
b. P 1,000,000
c. P 1,500,000
d. P 0

58. ABC Corp. has the following transactions during the year:
• Loans - Php 1,000,000, the DST due is Php 75,000 PHP1.50 on each 200 of the issue price
• Increase of capital stock - Php 5,000,000PHP2.00 on each 200 of the par value; or in case of no-par value based on the actual consideration
• Purchase of real property:
i. Selling price – Php 5,000,000
ii. Zonal value – Php 6,000,000 Php15. 00 for every Php1,000.00 of consideration or value, or factional part thereof
iii. FMV by assessor – Php 3,000,000
• Lease agreement – Php 5,000,000 PHP6.00 for the first 2,000 + PHP2.00 for every 1,000 thereafter

The total DST which should have been paid by the Company is:
a. Php 102,504.00
b. Php 152,504.00
c. Php 107,502.00
d. Php 157,502.00

Page 13 of 16
COMPREHENSIVE PROBLEMS

59. ABC Corp. is engaged in trading. The following information were disclosed in the VAT returns:

SALES
1st Quarter 2nd Quarter 3rd Quarter 4th Quarter
Domestic sales 3,000,000 4,000,000 3,000,000 5,000,000
VAT-exempt sales 2,000,000 1,000,000 1,000,000 -
Zero-rated sales 1,000,000 1,000,000 1,000,000 2,000,000

PURCHASES (all to VAT-registered suppliers)


1st Quarter 2nd Quarter 3rd Quarter 4th Quarter
Capital goods 500,000 500,000 500,000 1,000,000
Purchase of goods 5,000,000 1,000,000 1,000,000 1,000,000
Purchase of service 1,000,000 1,000,000 1,000,000 1,000,000
Imported goods 1,000,000 1,000,000 1,000,000 1,000,000

How much is the VAT Payable (Overpayment) in 1st Quarter?


a. (Php 240,000)
b. (Php 540,000)
c. (Php 900,000)
d. Php 360,000

60. How much is the VAT Payable (Overpayment) in 2nd Quarter?


a. (Php 110,000)
b. (Php 240,000)
c. Php 60,000
d. Php 130,000

61. How much is the VAT Payable (Overpayment) in 3rd Quarter?


a. (Php 86,000)
b. Php 24,000
c. Php 60,000
d. Php 130,000

62. How much is the VAT Payable (Overpayment) in 4th Quarter?


a. Php 34,000
b. Php 120,000
c. Php 600,000
d. Php 230,000

COMPREHENSIVE PROBLEMS

63. A non-VAT taxpayer’s 2021 records showed the following:


Gross sales
Quarters Head Office - Makati Branch 1 – Gapan City Branch 2 – Quezon
City
1st Quarter 200,000 100,000 100,000
2nd Quarter 300,000 100,000 100,000
3rd Quarter 500,000 100,000 100,000
4th Quarter 500,000 200,000 200,000
Total 1,500,000 500,000 500,000

Page 14 of 16
Assuming the taxpayer is Nicanor who opted for 8% preferential tax, how much is the percentage tax liability
for the 1st quarter?
a. Php 0
b. Php 4,000
c. Php 32,000
d. Php 12,000

64. Assuming the taxpayer is Nicanor who opted for Optional Standard Deduction for 1st Quarter, how much is
the Net Taxable Income for the taxable year 2021?
a. Php 1,500,000
b. Php 0
c. Php 900,000
d. Php 2,500,000

65. Assuming Nicanor opted for itemized deduction in the 1st quarter income tax return, how much is the total
percentage tax liability the taxable year 2021?
a. Php 25,000
b. Php 0
c. Php 250,000
d. Php 2,500

66. Assuming the taxpayer is ABC Corp, a domestic corporation which opted for Optional Standard Deduction
for 1st Quarter, how much is the income tax due for the taxable year 2021 assuming total assets excluding
the land amounted to Php 1,000,000 and the cost of sales amounted to Php 1,500,000?
a. Php 120,000
b. Php 375,000
c. Php 150,000
d. Php 300,000

67. Inday, the wife and the children of Nicanor provided the following data to you so you can help them file the
return and pay the corresponding estate tax, if any:

PROPERTIES
• Family home – Php 20,000,000
• Inherited 2,500 Shares in XYZ Corp. an unlisted company – 10,000,000
• Bank deposits – Php 5,000,000

Other information:
• Expenses and judicial expenses – Php 1,000,000

How much is the estate tax due?


a. Php 450,000
b. Php 750,000
c. Php 150,000
d. Php 390,000

68. The information provided by ABC Manufacturing Corp. disclosed the following:

INCOME STATEMENT
Gross Sales – Php 50M

Cost of goods/sales:
Direct materials – Php 10M
Direct labor – Php 10M
Manufacturing overhead – Php 5M

Page 15 of 16
Operating expenses:
Salaries and wages – Php 1M
Professional fees – Php 1M
Supplies – Php 1M
Depreciation – Php 1M
Training expenses* – Php 1M

BALANCE SHEET
Assets – Php 20M
Liability – Php 5M
SHE – Php 15M

*Requirements for additional deduction on training expenses are complied with.

Compute the income tax?


a. Php 4,875,000
b. Php 3,900,000
c. Php 5,000,000
d. Php 4,000,000

69. In 2021, ABC Corp. borrowed money from the bank in the amount of Php 1 Million with an interest rate of
10%. ABC Corp. has interest income from bank deposits in the amount of Php 50,000.00 from which final
tax was withheld. Assume ABC Corp.’s tax rate is 25%. How much is the allowable interest expense?
a. Php 90,000
b. Php 100,000
c. Php 83,500
d. Php 0

70. Which is FALSE?


a. BIR has 1 Commissioner and 6 Deputy Commissioners
b. BOC has 1 Commissioner and 6 Deputy Commissioners
c. PEZA Board has a total of 13 members including the chairman
d. BOI is comprised of 7 governors
e. BCDA is composed of 8 members of the board

End of Examination

(Pleases ignore the extra answer options in the answer sheet after number 70)

Thank you for participating in


Team PRTC Nationwide Online Open Final Pre-Board Examination for May 2022 LECPA!

Page 16 of 16
REGULATORY FRAMEWORK FOR BUSINESS TRANSACTIONS VILLEGAS/AGUILAR-APRADO
FINAL PRE-BOARD EXAMINATION April 25, 2022

Multiple Choice. Select the letter that corresponds to the best answer. This examination consists of
100 items only. The exam is good for three (3) hours. Good luck!

1. The following modes of extinguishment of obligations require two or more creditors, except:
a. Application of Payment
b. Legal Application of Payment
c. Payment by Cession
d. Dation in Payment

2. Below are instances when legal compensation is not allowed, except:


a. One of the debts arises from mutuum
b. One of the debts arises from commodatum
c. One of the debts arises from support by gratuitous title
d. One of the debts arises from civil liability arising from a criminal offense

3. S1 - Consent of parties is required in legal compensation. F


S2 - Consent of creditor is required in Substitution. T
S3 - In Dation in payment, the creditor becomes the owner of the property given. T
S4 - In Cession, creditors are only assignees. T
a. Only 1 statement is false
b. Only two statements are false
c. Only three statements are false
d. All statements are false

4. S1 – In Culpa Aquiliana, negligence is only incidental. F


S2 – In Culpa Contractual, ordinary diligence is available as a defense F
S3 – The Master and Servant Rule applies in Civil Negligence F
S4 – Proof beyond reasonable doubt is the evidence required in Culpa Criminal T
a. Only 1 statement is false
b. Only two statements are false
c. Only three statements are false
d. All statements are false

5. Instances when benefit to the creditor need not be proved if it redounded or not to him, except:
a. If after payment, the debtor acquired the right to collect from the guarantor
b. If after the payment, the third person acquires the creditor’s rights
c. If the creditor ratifies the payment to the third person
d. If the creditor’s conduct, the debtor has ben led to believe that third person had authority to receive
payment

6. The number of trustees in a non-stock corporation


a. Minimum of 1, maximum of 15
b. Minimum of 1, may be more than 15
c. Minimum of 2, maximum of 15
d. Minimum of 5, may be more than 15
e. Minimum of 5, maximum of 15

7. The number of board of directors in a stock corporation


a. Minimum of 1, maximum of 15
b. Minimum of 1, may be more than 15
c. Minimum of 2, maximum of 15
d. Minimum of 5, may be more than 15
e. Minimum of 5, maximum of 15

Page 1 of 15
8. Offenses against the confidentiality, integrity, and availability of computer data and system, except:
a. Illegal Access
b. Data Interference
c. System Interference
d. Misuse of Devices
e. Infringement

9. The provision aims at criminalizing the intentional hindering of the lawful use of computer systems
including telecommunications facilities by using or influencing computer data.
a. Illegal Access
b. Data Interference
c. System Interference
d. Misuse of Devices
e. Cyber squatting

10. A _______ grants property rights on an invention, allowing the holder to exclude others from making, selling,
or using the invention.
a. patent
b. trademark
c. utility model
d. copyright
e. industrial design

11. Alternative dispute resolutions (ADR) are recognized both in the Revised Corporation Code and the
Cooperative Law. This ADR is a process wherein the parties meet with a mutually selected impartial and
neutral person who assists them in the negotiation of their differences.
a. Negotiation
b. Mediation
c. Conciliation
d. Voluntary Arbitration
e. Mandatory Arbitration

12. S1 – The period to extend the life of the corporation if fixed is 5 years before its expiration. (3)
S2 – The period to extend the life of a cooperative is 5 years prior to its expiration.
S3 – The period to extend the patent is 6 months prior to its expiration. cannot be extended
a. S1 and S2 are false, S3 is true
b. S1 and S2 are true, S3 is false
c. S1 and S3 are false, S2 is true
d. S1 and S3 are true, S3 is false
e. All are false

13. The following entities are not allowed to be a One Person Corporation (OPC), except:
a. Banks
b. Pre-need
c. Insurance
d. Estate
e. Public and publicly listed companies

14. S1 – The bond of the single stockholder if he is likewise the treasurer is renewed every 3 years.
S2 – The vote to divide the Cooperative is ¾ of the members.
S3 – The limited partner may rightfully demand the return of his contribution after he has given a one (1) (6)
month notice in writing to all members if no time is specified in the certificate.
a. S1 and S2 are false, S3 is true
b. S1 and S2 are true, S3 is false
c. S1 and S3 are false, S2 is true
Page 2 of 15
d. S1 and S3 are true, S3 is false
e. All are false

15. The One Person Corporation is required to submit the following documents, except:
a. General Information Sheet
b. Audited Financial Statements
c. Disclosure of Self-Dealings and Related Party Transactions
d. Appointment Form
e. Compensation of director

16. Under the Revised Corporation Code, an Emergency Board may be created by a vote of _____________ to
(purpose) _________ and it will be temporarily filled from the ___________ of the corporation.
a. Majority vote of the OCS / prevent grave, substantial, and irreparable loss or damage to the corporation
/ stockholders
b. Majority vote of the BOD / vote on matter of paramount importance / officers
c. Unanimous vote of all the directors / prevent grave, substantial, and irreparable loss or damage to the
corporation / stockholders
d. Unanimous vote of the remaining directors / approve a self-dealing director / newly elected BOD
e. Unanimous vote of the remaining directors / prevent grave, substantial, and irreparable loss or damage
to the corporation / officers
(3)
17. S1 – The period to notify the creation of the emergency board is within 5 days from the creation of the board.
S2 – The period to report the election of the officers of the One Person Corporation is within 5 days from
their appointment.
S3 – The period to submit the certificate of stock for notation on the exercise of appraisal right is within 10
days after the demanding for payment.
a. S1 and S2 are false, S3 is true
b. S1 and S2 are true, S3 is false
c. S2 and S3 are false, S1 is true
d. S2 and S3 are true, S1 is false
e. All are false

18. S1 – The period for non-use of corporate charter is 5 years from the date of filing the articles of incorporation.
S2 – The period for non-use of a cooperative is 2 years from the issuance of the certificate of incorporation.
S3 – No damages can be recovered for acts of infringement of a patent committed more than three (3) years (4)
before the institution of the action for infringement.
a. S1 and S2 are false, S3 is true
b. S1 and S2 are true, S3 is false
c. S2 and S3 are false, S1 is true
d. S2 and S3 are true, S1 is false
e. All are false

19. A bound himself to deliver to B a 65-inch 2022 model of TV set, and the 18 cubic feet of White Westinghouse
refrigerator with motor no. WERT 385, which B saw in A’s store, and to repair B’s piano. A did not do any of
these.
S1 – A can be compelled to deliver the TV set.
S2 – A can be compelled to deliver the refrigerator.
S3 – A can be compelled to repair the piano.
a. S1 and S2 are false, S3 is true
b. S1 and S2 are true, S3 is false
c. S1 and S3 are false, S2 is true
d. S1 and S3 are true, S2 is false
e. All are false

20. A obligated himself to B for supplying textbook. It was agreed that the textbooks will be delivered by A on
November 20, 2021. Upon the arrival of the designated period for payment, is demand necessary in order
that A shall incur delay?
Page 3 of 15
a. Yes, following the general rule since there is no delay if there is no demand
b. Yes, since that is incumbent upon the rule on reciprocal obligation
c. No, demand is not necessary since there is a period designated delivered.
d. No, demand is not necessary since it is for the benefit of A

21. The debtor executed a promissory note promising to pay his obligation to the creditor as soon as he has
received funds derived from the sale of his property in certain place. What type of condition is present?
a. potestative
b. casual
c. mixed
d. negative

22. A sold a parcel of land to B for P200,000. In the deed of sale, there is a stipulation that the purchase price
shall be paid on a certain date and that in case of failure to pay on a certain date, then the contract shall be
automatically rescinded. The period arrived and B failed to pay as stipulated in the contract.
a. The contract is automatically rescinded as stipulated by the parties
b. The contract to be rescinded must need substantial breach to be rescinded.
c. The contract is not rescinded till after the arrival of the period designated and expiration of the grace
period as mandated by law.
d. The contract is not rescinded until there is notarial rescission.

23. S1 - An Asset Preservation Order is enforceable anywhere in the Philippines.


S2 - On the reporting of covered transactions, the law mandates that covered institutions shall report to the
AMLC all covered transactions within five (5) working days from occurrence thereof.
S3 - On record keeping under AMLA, the law states that all records of all transactions of covered institutions
shall be maintained and safely stored for five (5) years from the date of transactions.
a. Only 1 statement is false
b. Only 2 statements are false
c. All statements are false
d. All statements are true

24. S1 – The freeze order in AMLA shall be effective immediately within a period of 20 days.
S2 - On motion of the petitioner filed before the expiration of twenty days from issuance of a freeze order in
AMLA proceedings, the court may for good cause extend its effectivity for a period not exceeding six months.
S3 – The officers of the homeowners association shall be given fifteen (15) working days to refer the
application to the members of the association.
a. S1 and S2 are false, S3 is true
b. S1 and S2 are true, S3 is false
c. S1 and S3 are false, S2 is true
d. S1 and S3 are true, S2 is false
e. All are false

25. High Earning Corporation filed a complaint against five of its officers for violation of Section 31 of the
Revised Corporation Code. The corporation claimed that the said officers were guilty of advancing their
personal interests to the prejudice of the corporation, and that they were grossly negligent in handling its
affairs. Aside from documents and contracts, the corporation also submitted in evidence records of the
officers‘ U.S. Dollar deposits in several banks overseas - Boston Bank, Bank of Switzerland, and Bank of New
York. For their part, the officers filed a criminal complaint against the directors of High Earning Corporation
for violation of Republic Act No. 6426, otherwise known as the Foreign Currency Deposit Act of the
Philippines. The officers alleged that their bank deposits were illegally disclosed for want of a court order,
and that such deposits were not even the subject of the case against them. Under the Bank Secrecy Law,
can these foreign currency deposits be opened since they are subject matter of the litigation?
a. Yes, since the law is explicit on this matter that it is the subject matter of litigation.
b. Yes, if there is consent of the depositors
c. No, since the Bank Secrecy excludes FCDU accounts
d. No, a freeze order must be filed in the RTC to seize the FCDU accounts

Page 4 of 15
26. A private corporation was created by a special law. Later, the law creating it was declared invalid. May such
corporation claim to be a de facto corporation?
a. Yes, since from the inception the corporation was given juridical existence
b. No, where a private corporation is created under a special law, there is no attempt to incorporate a valid
corporation
c. No, since it is a public corporation and must be incorporated
d. Yes, it was a de jure corporation which gives it prima facie existence

27. Under the Articles of Incorporation of Manila Industrial Corp, its principal place of business shall be in Pasig,
MM. The principal corporate office is at the Ortigas Center, Pasig, MM while its factory processing leather
products is in Manila. The corporation holds its annual stockholders‘ meeting at the Manila Hotel in Manila
and its BOD meeting at a hotel in Makati MM. The by-laws are silent as to the place of meetings of the
stockholders and directors. Where should the meeting of the stockholders be held?
a. Pasig, MM only, being the principal place of business
b. Manila only, being the place of operation
c. Makati only, since it rests on the power of the BOD
d. Within Metro Manila, Metro Manila is considered a city or municipality

28. Stikki Cement Company was organized primarily for cement manufacturing. Anticipating substantial profits,
its President proposed that Stikki invest in a) a power plant project, b) a concrete road project, and c) quarry
operations for limestone in the manufacture of cement. What corporate approval or vote is needed for quarry
operations for limestones in the manufacture of the cement?
a. Majority of the BOD
b. Majority of the quorum of the BOD
c. Majority of the BOD and majority of the OCS
d. Majority of the BOD

29. The stockholders of People Power Inc (PPI) approved two resolutions in a special stockholders‘ meeting: a)
Resolution increasing the authorized capital stock of PPI; and b) Resolution authorizing the BOD to issue, for
cash payment, the new shares from the proposed capital stock increase in favor of outside investors who
are non-stockholders. The foregoing resolutions were approved by stockholders representing 99% of the
total outstanding capital stock. The sole dissenter was Jimmy Morato who owned 1% of the stock. Are the
resolutions binding on the corporation and its stockholders including Jimmy Morato, the dissenting
stockholder?
a. Yes, since only 1% OCS dissented to the approved act
b. No, since there was a lone dissenter and it must unanimously approved by the OCS
c. Yes, it is within the prerogative of the OCS to approve the intra vires act of the corporation.
d. No, While these resolutions were approved by the stockholders, the directors‘ approval, which is required
by law in such case, does not exist.

30. Which of the following corporate acts are valid, void, or voidable?
i) XL Foods Corporation, which is engaged in the fast-food business, entered into a contract with its
President Jose Cruz, whereby the latter would supply the corporation with its meat and poultry
requirements.
ii) The Board of Directors of XL Foods Corporation declared and paid cash dividends without approval of
the stockholders.
iii) XL Foods Corporation guaranteed the loan of its sister company XL Meat Products, Inc.
a. Valid, Void, Voidable
b. Void, Valid, Voidable
c. Voidable, Valid, Void
d. Valid, Void, Void
e. Voidable, Valid, Voidable

31. The BOD of a corporation declared due and payable all unpaid subscription to the capital stock. The lone
dissenting director failed to pay on due date, i.e., 19 Sept 2021, his unpaid subscription. Other than the shares
wherein he was unable to complete payment, he did not own any share in the corporation. On 23 Sept 2021,
he was informed by the BOD that, unless due payment is meanwhile received, he:
Page 5 of 15
a) could no longer serve as a director of the corporation forthwith:
b) would not be entitled to the cash and stock dividends which were declared and payable on 24 Sep 2021;
and
c) could not vote in the stockholders meeting scheduled to take place on 26 Sept 2021.
Was the action of the BOD on each of the foregoing matters valid?
a. Yes, yes, yes The period of 30 days within which the stockholder can pay the unpaid subscription had not yet expired
b. Yes, no, yes
The delinquency did not deprive thethe
stockholder of his
canright
pay to
thereceive
unpaiddividends declared.
had notHowever, the cash dividend declared
c. No, yes, no The period of 30 days within which stockholder
may be applied by the corporation to the unpaid subscription
subscription yet expired
d. No, no, yes
e. No, no, no

32. M and N were very good friends. N borrowed P10,000 from M. Because of their close relationship, the
promissory note executed by N provided that he would pay the loan “whenever his means permit.”
Subsequently, M and N quarreled. M now seeks to collect the loan because he is in dire need of money.
a. Yes, be can collect the money since there is no period agreed upon
b. Yes, since the conditional obligation is void being dependent on the debtor
c. No, he can only ask for rescission of the contract
d. No, he must first ask the court to fix the period

33. M and N were very good friends. N borrowed P10,000 from M. Because of their close relationship, the
promissory note executed by N provided that he would pay the loan “whenever his means permit.”
Subsequently, M and N quarreled. M now seeks to collect the loan because he is in dire need of money. What
kind of obligation is present?
a. Potestative
b. Causal
c. Mixed
d. Period
e. Alternative

34. A and B sold 1,000 sacks of rice to X and Y. On X’s request, A and B delivered the 1,000 sacks of rice to him.
X resold the rice without turning over any part of it or its price to Y. What kind of obligation is present?
a. Joint divisible obligation
b. Joint indivisible obligation
c. Solidary divisible obligation
d. Solidary indivisible obligation

35. A and B sold 1,000 sacks of rice to X and Y. On X’s request, A and B delivered the 1,000 sacks of rice to him.
X resold the rice without turning over any part of it or its price to Y. From whom will Y ask for delivery of his
share?
a. From A
b. From B
c. From A and B
d. From X
e. From A, B and X

36. A and B sold 1,000 sacks of rice to X and Y. On X’s request, A and B delivered the 1,000 sacks of rice to him.
X resold the rice without turning over any part of it or its price to Y. How much if any, can Y demand from the
person/s liable?
a. 0
b. 125
c. 250
d. 500
e. 1,000

37. The following contracts can be ratified, except:


a. Rescissible
b. Voidable
Page 6 of 15
c. Unenforceable
d. Unauthorized

38. In the following instances, the third person becomes part of the contract, except:
a. Contracts creating real rights
b. Interference in contractual relations
c. Delivery of the subject matter of the contract
d. Contracts to defraud creditors

39. I do that you may give


a. Facio ut des
b. Facio ut facias
c. Do ut des
d. Do ut facias

40. The contract shall be presumed to be an equitable mortgage in the following instances, except:
a. When the price of a sale with right to repurchase is unusually inadequate
b. When the vendor remains in possession as lessee or otherwise
c. When the vendor retains for himself a part of the purchase price
d. When the vendor binds himself to pay taxes on the thing sold.

41. The corporate term of a stock corporation is that which is stated in its Articles of Incorporation. It may be
extended or shortened by an amendment of the Articles when approved by majority of its Board of
Directors and:
a. approved and ratified by at least 2/3 of all stockholders.
b. approved by at least 2/3 of the stockholders representing the outstanding capital stock.
c. ratified by at least 2/3 of all stockholders.
d. ratified by at least 2/3 of the stockholders representing the outstanding capital stock.

42. P authorized A to sign a negotiable instrument in his (P’s) name. It reads: "Pay to B or order the sum of Php1
million. Signed, A (for and in behalf of P)." The instrument shows that it was drawn on P. B then indorsed to
C, C to D, and D to E. E then treated it as a bill of exchange. Is presentment for payment necessary in this
case?
a. No, since the drawer and drawee are the same person.
b. No, since the bill is non-negotiable, the drawer and drawee being the same person.
c. Yes, since the bill is payable to order, presentment is required for acceptance.
d. Yes, in order to hold all persons liable on the bill.

43. P sold to M a pair of gecko (tuko) for Php50,000.00. M then issued a promissory note to P promising to pay
the money within 90 days. Unknown to P and M, a law was passed a month before the sale that prohibits
and declares void any agreement to sell gecko in the country. If X acquired the note in good faith and for
value, may he enforce payment on it?
a. No, since the law declared void the contract on which the promissory note was founded.
b. No, since it was not X who bought the gecko.
c. Yes, since he is a holder in due course of a note which is distinct from the sale of gecko.
d. Yes, since he is a holder in due course and P and M were not aware of the law that prohibited the sale
of gecko.

44. The Board of Directors of XYZ Corp. unanimously passed a Board Resolution approving the taking of steps
that in reality amounted to willful tax evasion. On discovering this, the government filed tax evasion charges
against all the company’s members of the board of directors. The directors invoked the defense that they
have no personal liability, being mere directors of a fictional being. Are they correct?
a. No, since as a rule only natural persons like the members of the board of directors can commit
corporate crimes.
b. Yes, since it is the corporation that did not pay the tax and it has a personality distinct from its
directors.
c. Yes, since the directors officially and collectively performed acts that are imputable only to the
Page 7 of 15
corporation.
d. No, since the law makes directors of the corporation solidarily liable for gross negligence and bad faith
in the discharge of their duties.

45. Under the Intellectual Property Code, lectures, sermons, addresses or dissertations prepared for oral delivery,
whether or not reduced in writing or other material forms, are regarded as –
a. non-original works
b. original works
c. derivative works
d. not subject to protection

46. Which of the following indorsers expressly warrants in negotiating an instrument that 1) it is genuine and
true; 2) he has a good title to it; 3) all prior parties have capacity to negotiate; and 4) it is valid and subsisting
at the time of his indorsement?
a. The irregular indorser
b. The regular indorser
c. The general indorser
d. The qualified indorser

47. In case of disagreement between the corporation and a withdrawing stockholder who exercises his appraisal
right regarding the fair value of his shares, a three-member group shall by majority vote resolve the issue
with finality. May the wife of the withdrawing stockholder be named to the three member group?
a. No, the wife of the withdrawing shareholder is not a disinterested person.
b. Yes, since she could best protect her husband's shareholdings.
c. Yes, since the rules do not discriminate against wives.
d. No, since the stockholder himself should sit in the three-member group.

48. The Corporation Code sanctions a contract between two or more corporations which have interlocking
directors, provided there is no fraud that attends to it, and it is fair and reasonable under the circumstances.
The interest of an interlocking director in one corporation may be either substantial or nominal. It is nominal
if his interest:
a. does not exceed 25% of the outstanding capital stock
b. exceeds 25% of the outstanding capital stock
c. exceeds 20% of the outstanding capital stock
d. does not exceed 20% of the outstanding capital stock

49. Notice of dishonor is not required to be made in cases specified by law. One instance where such notice is
not necessary is when the indorser is the one to whom the instrument is supposed to be presented for
payment. The rationale here is that the indorser -
a. already knows of the dishonor and it makes no sense to notify him of it.
b. is bound to make the acceptance in all cases.
c. has no reason to expect the dishonor of the instrument.
d. must be made to account for all his actions.

50. X Corp., whose business purpose is to manufacture and sell vehicles, invested its funds in Y Corp., an
investment firm, through a resolution of its Board of Directors. The investment grew tremendously on
account of Y Corp.'s excellent business judgment. But a minority stockholder in X Corp. assails the
investment as ultra vires. Is he right and, if so, what is the status of the investment?
a. Yes, it is an ultra vires act of the corporation itself but voidable only, subject to stockholders’
ratification.
b. Yes, it is an ultra vires act of its Board of Directors and thus void.
c. Yes, it is an illegal act of its Board of Directors but voidable only, subject to stockholders’ ratification.
d. Yes, it is an ultra vires act of the corporation itself and, consequently, void.

51. In the election for the Board of Trustees of non-stock corporations, the right of the members of any class or
classes to vote may be limited, broadened, or denied and each member shall be entitled to one vote. This is
true -
Page 8 of 15
a. unless set aside by the members in plenary session.
b. in every case even if the Board of Trustees resolves otherwise.
c. unless otherwise provided in the Articles of Incorporation or in the By-laws.
d. in every case even if the majority of the members decide otherwise during the elections.

52. Under the Anti-Money Laundering Law, a covered institution is required to maintain a system of verifying the
true identity of their clients as well as persons purporting to act on behalf of
a. those doing business with such clients
b. unknown principals
c. the covered institution
d. such clients

53. ABC Corp. increased its capital stocks from Php10 Million to Php15 Million and, in the process, issued 1,000
new shares divided into Common Shares "B" and Common Shares "C." T, a stockholder owning 500 shares,
insists on buying the newly issued shares through a right of pre-emption. The company claims, however, that
its By-laws deny T any right of pre-emption. Is the corporation correct?
a. No, since the By-Laws cannot deny a shareholder his right of pre-emption.
b. Yes, but the denial of his pre-emptive right extends only to 500 shares.
c. Yes, since the denial of the right under the By-laws is binding on T.
d. No, since pre-emptive rights are governed by the articles of incorporation.

54. For purposes of determining violation of the provisions of Anti-Money Laundering Law, a transaction is
considered as a "Suspicious Transaction" with "Covered Institutions" regardless of the amount involved,
where which the following circumstances exist/s?
a. the amount involved is not commensurate with the client's business or financial capacity;
b. there is no underlying legal or trade obligation, purpose or economic justification;
c. client is not properly identified;
d. All of the above.

55. Which of the following is an exception to the secrecy of bank deposits and other laws which are in Philippine
Pesos, but not an exception to the secrecy of foreign currency deposits?
a. When the AMLC seeks to examine suspicious deposits or upon order of the court
b. Under the provisions of the Human Security Act
c. Upon inquiry in cases of impeachment
d. Written consent of depositor

56. Which phrase best completes the statement - When a debt is secured by a real estate mortgage, upon default
of the debtor?
a. the only remedy of the creditor is to foreclose the real estate mortgage;
b. another remedy is filing an action for collection and then foreclose if collection is not enough;
c. the creditor can foreclose the mortgage and demand collection for any deficiency;
d. None of the above.

57. X mortgaged her residential house and lot in favor of ABC Bank. X defaulted in her loan and so the bank
foreclosed the real estate mortgage on the residential house. Y then bought the residential house and lot
before the expiration of the redemption period. Can Y now take possession of the property?
a. No, because it is still covered by the redemption period and the purchaser is not yet entitled as a matter
of right to take possession of the property.
b. Yes, the purchaser is now entitled to the possession of the house.
c. No, because there is a need to talk to X to leave the house.
d. No, because Y was not the one who foreclosed the mortgage on the property.

58. Which phrase best completes the statement - The affidavit of good faith in a Deed of Chattel Mortgage is?
a. an oath where the parties swear that the mortgage is made for the purpose of securing the obligations
specified and that the obligation is just and valid;
b. an affidavit, the absence of which will vitiate the mortgage between the parties;
c. necessary only if the chattel being mortgaged are growing crops;
Page 9 of 15
d. a certification from the mortgagor that he is the mortgagor of the chattel.

59. Which phrase best completes the statement - To bind third parties, a chattel mortgage of shares of stock
must be registered?
a. with the Register of Deeds where the debtor resides;
b. with the Register of Deeds where the principal office of the corporation is;
c. in the Stock and Transfer Book of the corporation with the Corporate Secretary;
d. with the Register of Deeds where the debtor resides and the principal office of the corporation.

60. X is a depositor of AAA Bank. She has three (3) deposit accounts all under her name. One, in checking
account, one in saving account and another one in time deposit account. Each account has a balance of
Php250,000. AAA Bank became insolvent. Philippine Deposit Insurance Corporation was assigned as
receiver of the Bank. X therefore is unable to withdraw from all of the accounts.
She then filed her claims with the Philippine Deposit Insurance Corporation. Which statement is most
accurate?
a. X can claim a total of Php500,000 for all the three (3) accounts.
b. X can only claim from one (1) account of Php250,000.
c. X can claim a total of Php750,000 from all the three (3) accounts.
d. X cannot claim anything from any of the deposit accounts.

61. X is being charged for violation of Anti-Graft and Corrupt Practices because he is suspected of having
accumulated unexplained wealth. X maintains deposit accounts with ABC Bank. The Ombudsman filed
criminal cases against X before the Sandiganbayan. Can the Court issue subpoenas against ABC Bank to
produce all documents pertaining to all the deposit accounts of X?
a. Yes, because there is already a pending case and provided the subpoena must be specific as to which
account.
b. Yes, it is enough that the specific bank is identified.
c. No, because the issuance of the subpoena has no real legal basis.
d. Even without a subpoena, information about the deposit accounts of X can be submitted to the
Sandiganbayan because it will be used in a pending case.

62. XXX Bank Corporation and ZZZ Corporation were merged into XX ZZ Bank Corporation. So as not to create
any unnecessary conflict, all the former directors of both banks wanted to be appointed /elected as members
of the Board of Directors of the merged bank. Each bank used to have eleven (11) members of the board.
The maximum number of directors of the merged bank is -
a. 15
b. 22
c. 21
d. 11

63. X maintains a savings deposit in the amount of Php·1 Million with ABC Bank Corporation. X also has
obtained a loan from ABC Bank Corporation in the amount of Php1 Million. In case of default,
a. ABC Bank can set-off the loan from the savings account being maintained by X with ABC Bank
b. Set-off is not possible because legal compensation is not allowed in banking transaction
c. Deposit accounts are usually earmarked for specific purpose hence offsetting is not legally possible
d. Off -setting is not possible because the obligation of X is a "simple loan"

64. The government agency granted with the power of supervision and examination over banks and non-bank
financial institutions performing quasi-banking functions, to ensure that the conduct of its business is on a
sound financial basis that will provide continued solvency and liquidity is -
a. The Philippine Deposit Insurance Corporation
b. The Bangko Sentral ng Pilipinas
c. The Anti-Money Laundering Council
d. The Securities and Exchange Commission

65. X, who is the Executive Vice President of ABC Corporation, a listed company, can be held liable or guilty of
insider trading if, he -
Page 10 of 15
a. bought shares of ABC Corporation when it was planning to acquire another company to improve its asset
base, the news of which increased the price of the shares in the Stock Exchange
b. bought shares of XYC Corporation, a sister company of ABC Corporation when he learned that XYC
Corporation was about to also list its share in the Philippine Stock Exchange
c. bought shares of ZZZ Corporation when he learned that ABC Corporation would acquire ZZZ Corporation
d. All of the above

66. AAA Corporation is a foreign corporation that wants to operate a representative office here in the Philippines.
As required by the Corporation Code, there is a need to appoint a Resident Agent as a condition precedent
to the issuance of a license to transact business in the Philippines. After two (2) years, AAA Corporation
removed its Resident Agent and did not appoint anyone anymore. Which statement is the most accurate?
a. This can be a ground for revocation or suspension of its license to do business
b. There is no more effect in the license because anyway at the time of registration, a resident agent was
appointed
c. This can be a ground for suspension only
d. This will result in automatic revocation of its license to do business in the Philippines

67. X is a minority stockholder of CCC Corporation. Y is a member of the Board of Directors of CCC Corporation
and at the same time he is the President. X believes that Y is mismanaging CCC Corporation hence, as a
stockholder and in behalf of the other stockholders, he wanted to sue Y. Which statement is most accurate?
a. X can institute a derivative suit in behalf of himself as a stockholder.
b. A derivative suit must be instituted in behalf of the corporation.
c. Derivative suit is an exclusive remedy that X can institute.
d. Derivative suit is not the remedy in this situation.

68. If ABC Corporation will increase its authorized capital stock, the Corporation Code requires the approval of
the SEC and -
a. the approval of the majority of the Board of Directors only.
b. the approval of the majority of the stockholders and the Board of Directors.
c. the approval of 2/3 of the shareholders of the outstanding capital stock as well as the approval of the
Securities and Exchange Commission.
d. the approval of the majority of the Board of Directors and approval of the shareholders holding 2/3
share of the outstanding capital stock.

69. X subscribed 10,000 shares in the capital stocks of AAA Corporation. He paid 50% of the 10,000 shares. X
asked the Corporate Secretary to issue him the corresponding stock certificate representing the 50% of what
he already paid. The Corporate Secretary of the corporation refused.
Was the Corporate Secretary correct?
a. The Corporate Secretary is correct because the Revised Corporation Code provides that no certificate
of stock shall be issued to a subscriber until the shares as subscribed have been fully paid.
b. The Corporate Secretary cannot refuse because a Stock Certificate can be issued corresponding to the
percentage of shares which were paid.
c. The Corporate Secretary cannot refuse because a Certificate of Stock can be issued provided it is
indicated in the Certificate the actual percentage of what has been paid.
d. The Corporate Secretary cannot refuse because it is his legal duty to issue a stock certificate
corresponding to the number of shares actually subscribed regardless of the actual payment.

70. Which of the following is the least defective contract?


a. Rescissible
b. Voidable
c. Unenforceable
d. Void

71. In double sale of real property, priority is given to -


a. The one who first possessed the thing in good faith
b. The one to whom it was first sold in good faith
c. The one to whom who had the sale notarized in good faith
Page 11 of 15
d. The one to whom who had it registered in good faith

72. The penalty for violation of BP 22 is –


a. Imprisonment of not less than thirty (30) days but not more than one (1) year
b. Fine of not less than but not more than double the amount of the check, which shall in no case, exceed
Two Hundred Thousand (P200,000.00)
c. Both imprisonment and fine
d. Any of the above-mentioned

73. For applications or requests for license, clearance, permit, certification or authorization requiring the
approval of the local Sangguniang Bayan, Sangguniang Panlungsod, Sangguniang Panlalawigan, the
processing time is –
a. 3 working days
b. 7 working days
c. 20 working days
d. 45 working days

74. Applications or requests submitted by applicants or requesting parties of a government office or agency
which only require ministerial actions on the part of the public officer or employee, or that which present only
inconsequential issues for the resolution by an officer or employee of said government.
a. Simple Transactions
b. Compound Transactions
c. Complex Transactions
d. Highly Technical Transactions

75. The period to appeal the denial of the application for registration of a Cooperative from CDA to the Office
of the President is
a. 20 days
b. 30 days
c. 45 days
d. 60 days
e. 90 days

76. A partnership which compromises all the partners may acquire by their work or industry during the existence
of the partnership is
a. Universal partnership of present property c. Particular partnership
b. Universal partnership of profits d. General partnership

77. The common property of a universal partnership shall be


a. All the properties which shall belong to each of the partners after the constitution of the partnership.
b. All the properties which belong to each of the partners at the time of the constitution of the partnership.
c. All the properties which belong to each of the partners at the time of the constitution of the partnership
as well as the profits which they may acquire therewith.
d. All the properties which belong to each of the partners at the time of the constitution of the partnership
as well as properties which each may acquire thereafter.

78. Mr. AB offered in writing to sell his house and lot for P750,000.00 to Mr. CD on July 1, 2019. Mr. CD requested
Mr. AB to give him 60 days within which to raise the P750,000.00. On August 15, 2019, Mr. AB informed Mr.
CD that the price is raised and now at P1,000,000.00. Can Mr. CD compel Mr. AB to sell his house and lot at
P750,000.00 which was offered in writing by Mr. AB?
A. Yes, because Mr. AB is already estoppel by his written offer
B. Yes, because the 60 days offer has not yet expired
C. No, because Mr. CD has not accepted the offer of Mr. AB.
D. Yes, because there was already meeting of the minds

79. In distinguishing Earnest Money from Option Money, Earnest Money is


A. Given when there is no contract of sale
Page 12 of 15
B. Given only when there is a perfected contract of sale
C. Given to bind the offeror in a unilateral promise to sell or buy
D. Given as a separate consideration from the purchase price

80. Which of the following obligations of the vendor cannot be waived?


a. To allow the buyer to examine the goods sold.
b. To transfer ownership to the buyer.
c. To pay the expenses of the deed of sale.
d. To warrant the thing sold.

81. Three of the following contracts are inexistent and void from the beginning. Which is the exception?
a. That where one of the parties is incapable of giving his consent to a contract.
b. That whose purpose is contrary to law or public policy.
c. That which contemplates an impossible service.
d. That which is absolutely simulated or fictitious.

82. The following instances are subject to suspensive condition, except:


a. Contract to sell
b. Conventional redemption
c. Sale on trial
d. Sale on approval or satisfaction

83. Ralph and Vi orally agreed to form a partnership. Each contributed cash and personal properties worth
P10,000 to a common fund. But they did not register the partnership with the Securities and Exchange
Commission (SEC)
a. The partnership is still valid. c. The partnership is voidable.
b. The partnership is void. d. The partnership is unenforceable.

84. The following are judicial grounds for the dissolution of partnership, except:
a. insanity of a partner
b. Insolvency of any partner
c. Incapacity of a partner
d. Misconduct of a partner
e. Persistent breach of partnership agreement

85. The following are instances of acts of ownership in general partnership, except:
a. continue the partnership on the death, insanity or civil interdiction of a general partner
b. enter into a compromise concerning a partnership claim
c. submit a partnership or liability to arbitration
d. renounce a claim of the partnership
e. dispose of the goodwill of the business

86. The following are instances when the certificate in a limited partnership may be amended, except:
a. change in the name of the partnership
b. person is substituted as a limited partner
c. change in the character of the business of the partnership
d. false or erroneous statement in the certificate
e. all the limited partners cease to be such

87. The following are prohibited transactions in a limited partnership, except:


a. Receiving or holding as collateral security any partnership property
b. receiving any payment, conveyance or release from liability if it will prejudice the right of third person
c. granting loans to the partnership
d. Answer not given

88. A single-purpose cooperative may transform into a multi-purpose cooperative and may create subsidiaries
only after at least:
Page 13 of 15
a. 2 years of operation
b. 2 years from registration
c. 2 years from cooperation
d. 5 years from registration

89. No additional compensation other than per diems shall be paid to a director of a cooperative during of
existence of any cooperative.
a. First year c. First 3 years
b. First 2 years d. First 5 years

90. Upon the winding up of the cooperative affairs, any asset distributable to any creditor, shareholder or member
who is unknown or cannot be found shall be given to the:
a. Federation or union to which the cooperative is affiliated with
b. Government
c. Trustee
d. Any federation or union

91. Where no creditors are affected the cooperative may be voluntarily dissolved by vote of the board of directors
and by a resolution duly adopted by the affirmative vote of the members with voting rights, present and
constituting a quorum at a meeting to be held upon call of the directors.
a. Majority; 3/4 c. Majority, majority
b. 2/3, Majority d. Majority; 2/3

92. No member shall transfer his shares or interest in the cooperative or any part thereof unless he had held
such share capital contribution or interest for not less than:
a. One year c. 3 years
b. 2 years d. 5 years

93. Upon dissolution of a cooperative, what happens to the capital donated to said cooperative?
a. It is subject to escheat
b. It shall be divided among the members of the cooperative
c. It shall be returned to the donor
d. It shall be donated to another cooperative with a similar purpose

94. An elected officer of a cooperative may be removed by


a. BOD
b. General Assembly
c. BOD and General Assembly
d. Chairman of the Cooperative

95. The following are required to file their claims under the PDIC, except:
a. With valid deposit accounts with balances of more than P100,000.00;
b. With outstanding obligations with the closed bank either as borrower, co-maker, or as spouse of borrower;
c. With incomplete mailing address found in the bank records, or failed to update them through the MAUF
issued by the PDIC;
d. have not maintained the account under the name of business entities

96. Large scale in the Data Privacy Act is committed and the maximum penalty in the scale of penalties
respectively provided in the offenses shall be imposed when the personal information
___________________________ is harmed, affected or involved as the result of the above mentioned actions.
a. of at least 250 persons
b. of at least fifty (50) persons
c. of at least one hundred (100) persons
d. of at least one thousand (1,000) persons

97. The full report of the personal data breach must be submitted ________________

Page 14 of 15
a. within three (3) days
b. within five (5) days
c. within ten (10) days
d. within twenty (20) days
e. within seventy-two (72) hours

98. NO. 1: A contract entered into by a minor is void.

NO. 2: As a general rule, contracts where the amount involved P500 or more must be in writing to
be enforceable.

a. True, True c. False, True


b. False, False d. True, False

99. A limited partnership is formed if there is a substantial compliance in good faith with the requirements of the
law, except if this is violated:
a. character of the business
b. location of the principal place of businesss
c. the term which the partnership is to exist
d. the name of the partnership, adding thereto the word “Limited”

100. Where a partner has become insolvent or his estate is insolvent, the claims against his separate property
shall rank in the following order –
a. partnership creditors, separate creditors, partners by way of contribution
b. partners by way of contribution, separate creditors, partnership creditors
c. separate creditors, partners by way of contribution, partnership creditors
d. separate creditors, partnership creditors, partners by way of contribution

End of Examination

Thank you for participating in


Team PRTC Nationwide Online Open Final Pre-Board Examination for May 2022 LECPA!

Page 15 of 15
FINANCIAL ACCOUNTING AND REPORTING OCAMPO/OCAMPO
FINAL PRE-BOARD EXAMINATION April 26, 2022

Multiple Choice. Select the letter that corresponds to the best answer. This examination consists of
70 items only. (Please ignore the extra answer options in the answer sheet after number 70.) The exam
is good for three (3) hours. Good luck!

1. The Insurance Commission is represented in

I. FRSC
II. PIC
III.AASC

a. I, II and III
b. II and III only
c. II only
d. III only

2. Which of the following is the least authoritative?


a. IFRIC Interpretations
b. SIC Interpretations
c. PIC Q&As
d. None of these, all are equally authoritative

3. Bungee Co. is a calendar-year retailer. Its year-end physical count of inventory on hand did not consider the
effects of the following transactions:
• Goods with a cost of P50,000 were shipped by Bungee FOB shipping point on December 30 and were
tendered to and accepted by the buyer on Jan. 4.
• Goods with a cost of P40,000 were shipped FOB destination by a vendor on Dec. 30 and were tendered
to and accepted by Bungee on Jan. 4.
• Goods were sold on the installment basis by Bungee. Installment receivables representing sales of goods
with a cost of P30,000 were reported at year-end. Bungee retains title to such goods until full payment is
made.
• Goods with a cost of P20,000 were held on consignment for a vendor. These goods were excluded from
the count although they were sold in Jan.

If inventory based solely on the physical count of items on hand equaled P1,000,000. Bungee should report
inventory at year-end of
a. P1,000,000 c. P1,040,000 All items were properly excluded
b. P1,070,000 d. P1,020,000

4. Compassion Company reported inventories valued at P8,800,000 on Dec. 31. The following items were
included in this amount:
• Cryptocurrencies for investment purposes, P860,000.
• Lubricants that are consumed by the entity’s machinery in producing goods, P90,000.
• Materials in transit shipped FOB shipping point, P120,000.
• Finished goods in transit shipped FOB shipping point, P150,000.
• Advertising catalogs and shipping boxes, P30,000.
• Items of property, plant and equipment previously held for rental to others that are now held for sale in
the ordinary course of business, P240,000.

The adjusted inventories of Compassion Company at Dec. 31 should be


a. P7,430,000 c. P7,760,000
b. P7,520,000 d. P7,910,000

Page 1 of 18
5. A physical inventory taken on Dec. 31, 2022 resulted in an ending inventory of P1,440,000. Sun Company
suspects some inventory may have been taken by employees. To estimate the cost of missing inventory,
the following were gathered:

Inventory, Dec. 31, 2021 P1,280,000


Purchases during 2022 5,640,000
Cash sales during 2022 1,400,000
Shipment received on Dec. 26, 2022, included in physical inventory, but not
recorded as purchases 40,000
Deposits made with suppliers, entered as purchases. Goods were not received in
2022 80,000
Collections on accounts receivable, 2022 7,200,000
Accounts receivable, Jan. 1, 2022 1,000,000
Accounts receivable, Dec. 31, 2022 1,200,000
Gross profit percentage on sales 40%

At Dec. 31, 2022 what is the estimated cost of missing inventory?


a. P200,000 c. P240,000
b. P160,000 d. P320,000

Use the following information for the next two questions.

Gatas Dairy produces milk to sell to local and national ice cream producers. Gatas Dairy began operations on
Jan. 1, 2022 by purchasing 840 milk cows for P1,176,000. The company controller had the following information
available at year end relating to the cows:

Carrying value, Jan. 1, 2022 P1,176,000


Increase in fair value due to growth
and price changes 365,000
Decrease in fair value due to harvest 42,000
Milk harvested during 2022 but not yet sold 54,000

6. At Dec. 31, 2022, what is the value of the milking cows on Gatas Dairy’s statement of financial position?
a. P1,583,000 c. P1,499,000
b. P1,553,000 d. P1,445,000

7. On Gatas Dairy’s income statement for the year ending Dec. 31, 2022, what amount of unrealized gain on
biological assets will be reported?
a. P461,000 c. P377,000
b. P407,000 d. P323,000

Information relevant to five different entities follows.

• Newcastle Corp. uses many kinds of machines in its operations. It constructs some of these machines itself
and acquires others from the manufacturers. The following information relates to machine A that it has
recorded during the current year.

Cash paid for equipment, including VAT of P9,600 P89,600


Costs of transporting machine - insurance and transport 3,000
Labor costs of installation by expert fitter 5,000
Labor costs of testing equipment 4,000
Insurance costs for current year 1,500
Costs of training for personnel who will use the machine 2,500
Costs of safety rails and platforms surrounding machine 6,000
Costs of water devices to keep machine cool 8,000
Page 2 of 18
Costs of adjustments to machine to make it operate more efficiently 7,500

• In Jan. 2022, Utah Corp. entered into a contract to acquire a new machine for its factory. The machine, which
had a cash price of P2,000,000, was paid for as follows:

Down payment P 300,000


5,000 ordinary shares of Utah with an agreed-upon value of P370 per share 1,850,000
P2,150,000

Prior to the machine's use, installation costs of P70,000 were incurred. The machine has an estimated useful
life of 10 years and an estimated salvage value of P100,000. The straight-line method of depreciation is used.

• On Mar. 31, 2022, Nathaniel Corp. traded in an old machine having a carrying amount of P168,000, and paid
a cash difference of P60,000 for a new machine having a total cash price of P205,000. The cash flows from
the new machine are expected to be significantly different than the cash flows from the old machine.

• Gentleness Corp. takes a full year’s depreciation in the year of an assets acquisition, and no depreciation in
the year of disposition. Data relating to one depreciable asset acquired in 2020, with residual value of
P900,000 and estimated useful life of 8 years, at Dec. 31, 2021 are:

Cost P9,900,000
Accumulated depreciation 4,331,250

• At Jan. 1, 2022, the revaluation surplus of Bloxden was P1,257,000. This was in respect of the company’s
head office. During the year to Dec. 31, 2022, the value of the head office increased by a further P82,000. In
the same period, the company’s factory suffered an impairment of P90,000.

8. Determine the amount at which machine A should be recorded in the records of Newcastle Corp.
a. P105,500 c. P116,000
b. P113,500 d. P121,500

9. The depreciation on the machine to be recognized by Utah Corp. in 2022 is


a. P212,000 c. P1820,000
b. P190,000 d. P197,000

10. On Mar. 31, 2022, what amount of loss should Nathaniel recognize on the exchange?
a. P60,000 c. P23,000
b. P37,000 d. P 0

11. Using the same depreciation method in 2020 and 2021, how much depreciation should Gentleness record in
2022 for this asset?
a. P1,392,188 c. P1,856,250
b. P1,500,000 d. P2,475,000

12. What is the value of Bloxden’s revaluation surplus at Dec. 31, 2022?
a. P1,167,000 c. P1,257,000
b. P1,249,000 d. P1,339,000

Page 3 of 18
Information relevant to five different entities follows.

• The Niagara Company owns three properties which are classified as investment properties according to PAS40
Investment property. Details of the properties are given below (amounts in thousands):

Initial Fair value at Fair value at


cost Dec. 31, 2021 Dec. 31, 2022
Property (1) 270 320 350 30
Property (2) 345 305 285 (20)
Property (3) 330 385 360 (25)

Each property was acquired in 2018 with a useful life of 50 years. The company's accounting policy is to use
the fair value model for investment properties.

• Hall Co. incurred research and development costs in 2022 as follows:

Materials used in research and development projects P 450,000


Equipment acquired that will have alternate future uses in future research and
development projects 3,000,000
Depreciation for 2022 on above equipment 300,000
Personnel costs of persons involved in research and development projects 750,000
Consulting fees paid to outsiders for research and development projects 300,000
Indirect costs reasonably allocable to research and development projects 225,000
P5,025,000

Assume economic viability has not been achieved.

• Mini Corp. acquires a patent from Maxi Co. in exchange for 2,500 shares of Mini Corp.’s P5 par value ordinary
shares and P75,000 cash. When the patent was initially issued to Maxi Co., Mini Corp.’s shares were selling
at P7.50 per share. When Mini Corp. acquired the patent, its shares were selling for P9 a share.

• Yakal Exploration Co. purchased in 2020 a property that contained mineral deposit for P4,500,000. Estimated
recovery was 1,000,000 metric tons of deposits. Development costs of P150,000 were also incurred in the
same year. The mining property was expected to be worth P600,000 after the mineral deposits had all be
removed. During 2021, the company extracted and sold 100,000 metric tons of minerals. Further
development costs of P75,000 were incurred in 2022, and the estimate of total recoverable deposits
(including the amount extracted in 2019) was revised to 925,000 metric tons. During 2022, the company
recovered 150,000 metric tons.

• Entity S reported an impairment loss of P1,000,000 in profit or loss for the year 2021. This loss was related
to an equipment which was acquired on Jan. 1, 2020 for P6,250,000, useful life of 10 years and no residual
value. The straight-line method is used in recording depreciation of this asset. On the Dec. 31, 2021
statement of financial position, the entity reported this asset at P4,000,000 (the recoverable amount on that
date). The entity revised the remaining life of the asset to 5 years starting Jan. 1, 2022.

On Dec. 31, 2022, the entity decided to measure the asset using revaluation model. This asset was then
appraised at a fair value of P4,500,000.

13. What is the gain or loss to be recognized in Niagara's profit or loss for the year ending Dec. 31, 2022?
a. P18,900 loss c. P30,000 gain
b. P15,000 loss d. P45,000 loss

14. The amount of research and development costs charged to Hall's 2022 income statement should be
a. P1,500,000 c. P2,025,000
b. P1,900,000 d. P4,500,000

Page 4 of 18
15. Mini Corp. should record the patent at what amount?
a. P87,500 c. P97,500
b. P93,750 d. P75,000

16. Yakal’s depletion for the year 2022 is


a. P603,658 c. P676,500
b. P618,750 d. P750,000

17. The reversal of impairment loss to be recognized by Entity S in 2022 profit or loss is
a. P 825,000 c. P1,175,000
b. P1,000,000 d. P1,300,000

18. The Angelbert Company accounts for non-current assets using the revaluation model. On June 30, 2021,
Angelbert classified a freehold property as held for sale in accordance with PFRS5. At that date the property's
carrying amount was P290,000 and the balance on the revaluation reserve was P20,000. At that date its fair
value was estimated at P330,000 and the costs to sell at P20,000. At Dec. 31, 2021, the property's fair value
was estimated at P325,000 and the costs to sell at P25,000.

If the asset was sold for a net proceeds of P285,000 in 2022, what amount should be included as loss on
disposal in the entity's statement of comprehensive income for the year ended Dec. 31, 2022?
a. P 5,000 c. P25,000
b. P15,000 d. P35,000

19. Which statement is incorrect regarding government grants?


a. A government grant is not recognized until there is reasonable assurance that the entity will comply with
the conditions attaching to it, and that the grant will be received.
b. Receipt of a grant does not of itself provide conclusive evidence that the conditions attaching to the grant
have been or will be fulfilled.
c. The manner in which a grant is received affects the accounting method to be adopted in regard to the
grant.
d. A forgivable loan from government is treated as a government grant when there is reasonable assurance
that the entity will meet the terms for forgiveness of the loan.

20. Which statement is incorrect regarding accounting for borrowing costs?


a. Borrowing costs do not include actual or imputed cost of equity, including preferred capital not classified
as a liability.
b. Borrowing costs that are directly attributable to the acquisition, construction or production of a qualifying
asset shall be recognized as part of the cost of asset.
c. The borrowing costs that are directly attributable to the acquisition, construction or production of a
qualifying asset are those borrowing costs that would have been avoided if the expenditure on the
qualifying asset had not been made.
d. Capitalization of borrowing costs continues if minor modifications, such as the decoration of a property
to the user’s specification, are still outstanding.

21. Aries Company started construction on a building on Jan. 1 of the current year and completed construction
on Dec. 31 of the same year. Aries had only two interest notes outstanding during the year, and both of these
notes were outstanding for all 12 months of the year. The following information is available:

Average accumulated expenditures P250,000


Ending balance in construction in progress before capitalization of interest 360,000
6 percent note incurred specifically for the project 150,000
9 percent long-term note 500,000

What amount of interest should Aries capitalize for the current year?
a. P27,900 c. P18,000
b. P22,500 d. P15,000
Page 5 of 18
22. An entity holds a demand deposit whose terms and conditions do not prevent the entity from accessing the
amounts held in it (that is, were the entity to request any amount from the deposit, it would receive that
amount on demand). The entity also has a contractual obligation with a third party to keep a specified
amount of cash in that separate demand deposit and to use the cash only for specified purposes. If the entity
were to use the amounts held in the demand deposit for purposes other than those agreed with the third
party, the entity would be in breach of its contractual obligation. How should the entity present the demand
deposit in its statement of financial position?
a. As cash and cash equivalents
b. As other current assets
c. As other non-current assets
d. As an offset from the obligation to the third party

23. Las Piñas Corporation’s accountant is attempting to determine the amount of cash to be reported on its Dec.
31, 2022 statement of financial position. The following information is provided:

a. Commercial savings account of P1,200,000 and a commercial checking account balance of P1,800,000
are held at PS Bank.
b. Travel advances of P360,000 for executive travel for the first quarter of the next year (employee to
reimburse through salary deduction).
c. A separate cash fund in the amount of P3,000,000 is restricted for the retirement of a long-term debt.
d. Petty cash fund of P10,000.
e. An I.O.U. from a company officer in the amount of P40,000.
f. A bank overdraft of P250,000 has occurred at one of the banks the company uses to deposit its cash
receipts. At the present time, the company has no deposits at this bank.
g. The company has two certificates of deposit, each totaling P1,000,000. These certificates of deposit
have maturity of 120 days.
h. Las Piñas has received a check dated Jan. 2, 2023 in the amount of P150,000.
i. Las Piñas has agreed to maintain a cash balance of P200,000 at all times at PS Bank to ensure future
credit availability.
j. Bills and coins on hand amounted to P15,000.
k. Cryptocurrencies valued at P100,000.

How much will be reported as cash and cash equivalents at Dec. 31, 2022?
a. P2,575,000 c. P3,025,000
b. P2,825,000 d. P5,025,000

24. Woody Company's accountant is preparing its October bank reconciliation and has collected the following
data:

Per Books Per Bank


Oct. 1 balance P11,600 P10,000
Oct. deposits 24,600 21,200
Oct. checks 27,800 29,000
Note collected (includes 10% interest) - 4,400
Oct. service charge - 20
Oct. 31 balance 8,400 6,580

Additionally, deposits in transit and outstanding checks from September's reconciliation were, P4,400 and
P2,800 respectively.

The correct balance for cash at Oct. 31 should be:


a. P13,980 c. P11,180
b. P12,780 d. P10,960

Page 6 of 18
25. Lemonade Corp. had a 1/1/22 balance in the Allowance for Doubtful Accounts of P10,000. During 2022, it
wrote off P7,200 of accounts and collected P2,100 on accounts previously written off. The balance in
Accounts Receivable was P200,000 at 1/1 and P240,000 at 12/31. At 12/31/22, Lemonade estimates that
5% of accounts receivable will prove to be uncollectible. What is Doubtful Accounts Expense for 2022?
a. P2,000 c. P 9,200
b. P7,100 d. P12,000

26. Rafaela Corp. has P3,000,000 note receivable from sale of equipment bearing interest at 12% per annum.
The note is dated June 1, 2021. The note is payable in 3 annual installments of P1,000,000 plus interest on
the unpaid balance every June 1. The initial principal and interest payment was made on June 1, 2022.

The interest income for 2022 is


a. P360,000 c. P300,000
b. P310,000 d. P290,000

27. On July 1, 2022, Eudora Co. sold a machine costing P500,000 with accumulated depreciation of P380,000
on the date of sale. Eudora received as consideration for the sale, a P300,000 noninterest-bearing note, due
July 1, 2025. There was no established exchange price for the equipment and the note had no ready market.
The prevailing rate of interest for a note of this type at July 1, 2022 was 12% and 13% on Dec. 31, 2022. In
relation to this transaction, the total income to be recognized in Eudora’s 2022 profit or loss is
a. P180,000 c. P106,352
b. P119,165 d. P101,445

28. On Dec. 1 2022, an entity sells goods to a customer and recognizes a trade receivable of P100,000. On Dec.
31, 2022 (the entity’s reporting date), the customer notifies the entity that it has initiated a payment of
P100,000 via electronic payment system to settle the trade receivable. The entity received P100,000 into its
bank account on Jan. 2, 2023. When should the entity derecognize the trade receivable?
a. Dec. 1, 2022
b. Dec. 31, 2022
c. Jan. 2, 2023
d. Either b or c, whichever provides more favorable financial position

29. When bonds are acquired at a premium and the effective interest method is used, at each interest payment
date, the interest income:
a. Remains constant.
b. Is equal to the change in book value.
c. Increases.
d. Decreases.

30. The following information pertains to investment in ordinary shares of Entity X:


• Dec. 1, 2021 - acquired ordinary shares of Entity A for P200,000. The transaction costs paid were P3,000.
• Dec. 31, 2021 - the fair value of the investment was P208,000 and the transaction costs that would be
incurred on sale were estimated at P3,200.
• Mar. 31, 2022 - sold the investment for its fair value of P220,000. The transaction costs paid were P3,300.

Compute for the gain on sale of investment on Mar. 31, 2022.


a. P 8,700 c. P12,000
b. P11,900 d. P15,200

31. On Dec. 28, 2022, Eric Company commits itself to purchase a financial asset to be classified as held for
trading for P500,000, its fair value on commitment (trade) date. This security has a fair value of P505,000
and P510,000 on Dec. 31, 2022 (Eric's financial year-end), and January 5, 2023 (settlement date),
respectively.

If Eric applies the settlement date accounting method to account for regular-way purchases of its securities,
how much should be recognized as fair value adjustment gain in its 2022 profit or loss?
Page 7 of 18
A. P15,000 C. P5,000
B. P10,000 D. Nil

32. Minalin, Inc. acquired 30% of Momo Corp.'s ordinary shares on Jan. 1, 2022 for P360,000. During 2022, Momo
reported profit of P150,000 and paid dividends of P90,000. Minalin's 30% interest in Momo gives Minalin the
ability to exercise significant influence over Momo's operating and financial policies.

On Dec. 31, 2022, Minalin classified 40% of the investment as held for sale in accordance with PFRS 5. Total
fair value of the investment was P390,000 on that date. Minalin normally incurs costs to sell equal to 5% of
the selling price.

Determine the amount to be reported for the ‘held for sale’ portion of the investment.
a. P148,200 c. P156,000
b. P151,200 d. P226,800

33. Casiguran Corp. took out a P5,000,000 insurance policy on the life of its president on Jan. 1, 2013. Given
below are data on this policy:

2021 2022
Annual dividend P 3,880 P 4,210
Cash surrender value, 12/31 138,030 189,350
Annual premium 121,040 121,040

The life insurance expense for Casiguran Corp. for 2022 would be
a. P64,100 c. P116,830
b. P65,510 d. P121,040

34. Which statement is correct regarding derivatives?


a. An entity shall recognize derivatives using settlement date accounting.
b. Derivatives are usually classified as current in the statement of financial position.
c. Derivatives are initially recognized at its fair value plus transaction costs.
d. Derivatives can be subsequently measured at fair value through other comprehensive income.

35. A lessee decreased the carrying amount of a right-of-use asset and recognized a gain in profit or loss. This
is the accounting for which type of lease modification?
a. A lease modification accounted for as a separate lease.
b. A lease modification that decreased the scope of the lease and accounted for as a separate lease.
c. A lease modification that decreased the scope of the lease and not accounted for as a separate lease.
d. A lease modification that increased the scope of the lease and not accounted for as a separate lease.

36. A seller-lessee continued to recognize the transferred asset and recognized a financial liability equal to the
transfer proceeds. This is the accounting for which type of sale and leaseback?
a. A sale and leaseback that satisfies the requirements of PFRS 15 to be accounted for as a sale.
b. A sale and leaseback that satisfies the requirements of PFRS 15 to be accounted for as a sale but the
fair value of the consideration for the sale of an asset does not equal the fair value of the asset.
c. A sale and leaseback that satisfies the requirements of PFRS 15 to be accounted for as a sale but the
payments for the lease are not at market rates.
d. A sale and leaseback that does not satisfy the requirements of PFRS 15 to be accounted for as a sale.

Page 8 of 18
Information relevant to five different entities follows.

• The following information relates to the defined benefit pension plan of the Lupet Corp. for the year ended
Dec. 31, 2022:

Projected benefit obligation, Jan. 1 P9,000,000

Fair value of plan assets, Jan. 1 8,000,000


Service cost 1,000,000
Expected return on plan assets 900,000
Actual return on plan assets 850,000
Employer contributions 800,000
Benefits paid to retirees 780,000
Increase in projected benefit obligation due to changes in actuarial assumptions 160,000
Discount rate 10%

• At the beginning of year 1, Addo Corp. grants 100 share options to each of its 200 employees. Each grant
is conditional upon the employee remaining in service over the next three years. The entity estimates that
the fair value of each option is P21. On the basis of a weighted average probability, the entity estimates that
60 employees will leave during the three-year period and therefore forfeit their rights to the share options.

Suppose that 15 employees leave during year 1. Also suppose that by the end of year 1, the entity’s share
price has dropped, and the entity reprices its share options, and that the repriced share options vest at the
end of year 3. The entity estimates that a further 35 employees will leave during years 2 and 3. During year
2, a further 10 employees leave, and the entity estimates that a further 10 employees will leave during year
3. During year 3, a total of 8 employees leave.

The entity estimates that, at the date of repricing, the fair value of each of the original share options granted
(ie before taking into account the repricing) is P10 and that the fair value of each repriced share option is
P13.

• In 2023, before the Entity P’s 2022 financial statements were approved for issue, a class action lawsuit was
filed against the entity. The lawsuit seeks compensation for a community experiencing health problems
allegedly caused by pollution from the entity’s plant. Legal counsel advised management that there is a 30
per cent chance that the action will be successful. If successful, the court is likely to award the community
compensation of between P1,000,000 and P2,000,000.

• The accounting profit before tax for the year ended Dec. 31, 2022 for Oki Corp. amounted to P18,500 and
included:

Depreciation – motor vehicle (25%) P 4,500


Depreciation - equipment (20%) 20,000
Rent revenue 16,000
Royalty revenue (exempt from tax) 5,000
Doubtful debts expense 2,300
Entertainment expense (non-deductible) 1,500
Proceeds on sale of equipment 19,000
Carrying amount of equipment sold 18,000
Annual leave expense 5,000

The draft statement of financial position at Dec. 31, 2022 contained the following assets and liabilities:

2022 2021
Assets
Cash P 11,500 P 9,500
Receivables 12,000 14,000
Allow. for doubtful debts (3,000) (2,500)
Page 9 of 18
2022 2021
Inventory 19,000 21,500
Rent receivable 2,800 2,400
Motor vehicle 18,000 18,000
Acc. Dep. - motor vehicle (15,750) (11,250)
Equipment 100,000 130,000
Acc. Dep. - equipment (60,000) (52,000)
Deferred tax asset ? 5,550
P135,200

Liabilities
Accounts payable 15,655 21,500
Provision for annual leave 4,500 6,000
Current tax liability ? 7,600
Deferred tax liability ? 2,745
37,845

Additional information
a) The entity can claim a deduction of P15,000 (15%) for depreciation on equipment, but the motor vehicle
is fully depreciated for tax purposes.
b) The equipment sold during the year had been purchased for P30,000 two years before the date of sale.
c) The entity is subject to 30% tax rate.

37. The prepaid/accrued benefit expense of Lufet Corp. at Dec. 31, 2022 is
a. P2,100,000 c. P1,300,000
b. P2,210,000 d. P1,410,000

38. The amount to be recognized as expense by Addo corp. in year 3 is


a. P400,800 c. P150,750
b. P136,800 d. P145,050

39. In its financial statements for the year ended Dec. 31, 2022, the Entity P should recognize a liability for the
lawsuit of
a. P2,000,000 c. P1,000,000
b. P1,500,000 d. Nil

40. Oki Corp.’s current tax expense for 2022 is


a. P6,030 c. P7,500
b. P6,930 d. P8,040

41. Oki Corp.’s deferred tax expense (benefit) for 2022 is


a. P6,570 c. P(2,430)
b. P(3,270) d. P(1,080)

Use the following information for the next two questions.

An entity has a nuclear power plant and a related decommissioning liability. The nuclear power plant started
operating on Jan. 1, 2019. The plant has a useful life of 40 years. Its initial cost was P120 million, this included
an amount for decommissioning costs of P10 million, which represented estimated cash flows payable in 40
years discounted at a risk-adjusted rate of 5 per cent. The entity’s financial year ends on Dec. 31.

The entity adopts the revaluation model on Dec. 31, 2021. A market-based discounted cash flow valuation of
P115 million is obtained at Dec. 31, 2021. It includes an allowance of P11.6 million for decommissioning costs,
which represents no change to the original estimate, after the unwinding of three years’ discount.

Page 10 of 18
On Dec. 31, 2022, the decommissioning liability (before any adjustment) is P12.2 million and the discount rate
has not changed. However, on that date, the entity estimates that, as a result of technological advances, the
present value of the decommissioning liability has decreased by P5 million.

The entity decides that a full valuation of the asset is needed at Dec. 31, 2022, in order to ensure that the carrying
amount does not differ materially from fair value. The asset is now valued at P107 million, which is net of an
allowance of P7.2 million for the reduced decommissioning obligation that should be recognized as a separate
liability.
The entity does not transfer realized surplus directly to retained earnings.

42. The entity should report revaluation surplus as of Dec. 31, 2021 at
a. P15.6 million c. P4 million
b. P13.25 million d. P1.65 million

43. The entity should report revaluation surplus as of Dec. 31, 2022 at
a. P8.358 million c. P0.108 million
b. P11.622 million d. Nil

Information relevant to five different entities follows.

• Entity Z was incorporated on June 1, 2022 with an authorized 200,000, no-par, ordinary shares, stated value
P10 and 10,000, 9% par value P30, preference shares. Transactions affecting company’s equity as of June
30, 2022 were as follows:

June 1 50,000 ordinary shares were issued at P10.


June 5 Assets with a total appraised value of P600,000 were acquired in exchange for 50,000 ordinary
shares.
June 15 Subscriptions were received for 100,000 ordinary shares at P15 and for 5,000 preference
shares at P35.
June 25 Payments in full for the ordinary and preference shares subscribed June 15 were received and
the corresponding shares were issued.

• The capital accounts of Kamprad, Inc. on Dec. 31, 2021, were as follows:

Preference share capital, 20,000 shares, P20 par P 400,000


Share premium - preference 160,000
Ordinary share capital, 50,000 shares, P80 par 4,000,000
Share premium – ordinary 600,000
Retained earnings 360,000

During the year ending Dec. 31, 2022, the following summarizes the transactions affecting the shareholders’
equity

April 30 - 1,000 preference shares were retired at P25 per share.


June 15 - 2,000 treasury shares, ordinary, were purchased at P85 per share
June 30 - A two-for-one ordinary share split was declared.
July 31 - 800 treasury shares were reissued at P50 per share.
Dec. 31 – Profit for 2022 was P300,000.

• The Retained Earnings account of Lester Corp. for the year 2022 consists of the following items:

Debit Credit
Balance, Jan. 1, 2022 P112,500
Write-off of organization costs P 6,000
Excess of issuing price of share capital over par value 24,000

Page 11 of 18
Debit Credit
Loss on the sale of equipment 2,500
Correction of error of prior year 10,500
Gain on sale of treasury shares 3,500
Cash and share dividends 60,000
Net income for the year 58,500
Balance, Dec. 31, 2022 119,500 .
P198,500 P198,500

• The shareholders’ equity for the Fiesta Foods, Inc. at the end of the reporting period follows:

12% Preference share capital, P100 par, 20,000 shares P2,000,000


Ordinary share capital, P25 par, 200,000 shares 5,000,000
Share premium 500,000
Retained earnings 750,000
Total shareholders’ equity P8,250,000

Preference shares have a liquidation value of P110; shares are cumulative, with dividends in arrears for 3
years including the current year and fully payable in the event of liquidation.

• For the year ended Dec. 31, 2021, Orchard, Inc. had per share earnings of P4.80. Orchard's outstanding shares
for the 2021 consisted of P2,000,000 of 10% cumulative preference shares with P100 par value and
1,000,000 ordinary shares. On Jan. 1, 2022, the ordinary shares were split 3 for 1, and the entity redeemed
one-half of the preference shares at par value. Orchard's profit for the year ended Dec. 31, 2022 was 10%
higher than in 2021.

44. The total shareholders’ equity as of June 30, 2022 is


a. P2,875,000 c. P2,750,000
b. P2,300,000 d. P2,775,000

45. What was the total share premium of Kamprad, Inc on Dec. 31, 2022?
a. P760,000 c. P755,000
b. P766,000 d. P761,000

46. The correct balance of Lester Corp.’s retained earnings on Dec. 31, 2022 is
a. P119,500 c. P94,500
b. P100,500 d. P92,000

47. The book value of an ordinary share of Fiesta Foods, Inc. is


a. P28.85 c. P26.65
b. P25.35 d. P22.90

48. Orchard, Inc.’s earnings per share for the year ended Dec. 31, 2022 is
a. P1.73 c. P1.80
b. P1.77 d. P2.70

49. XYZ issues a P10 million note with no maturity date that is redeemable at the discretion of the issuer, but
can be converted by the holder for 10 million ordinary shares of XYZ at any time. According to PAS 32, XYZ
should classify the instrument as which one of the following?
a. Equity because the holder is exposed to changes in the fair value of the issuer’s shares
b. A liability because the holder is exposed to changes in the fair value of the issuer’s shares
c. Equity because the holder is not exposed to changes in the fair value of the issuer’s shares
d. A liability because the holder is not exposed to changes in the fair value of the issuer’s shares

50. An entity discovered errors in its ending inventory for the year ended Dec. 31, 2022. The error was discovered
in early 2023, after the books were closed. Some inventory in the amount of P12,000 was counted twice and
inventory valued at P5,000 was excluded from the inventory count because it was in transit (with terms FOB
Page 12 of 18
shipping point). The tax rate is 30%. Which of the following would be included in the correcting journal entry
to be done in 2023?
a. Debit deferred income tax P1,500
b. Debit retained earnings P4,900
c. Debit cost of goods sold P12,000
d. Credit inventory for P12,000

51. Information is not capable of making a difference in a decision if some users


a. Choose not to take advantage of it.
b. Are already aware of it from other sources.
c. Either a or b.
d. Neither a nor b.

52. Which one of the following is required to be presented as minimum information on the face of the Statement
of Financial Position, according to PAS 1?
a. Cash in bank
b. Finished goods inventory
c. Accounts receivable
d. Investments accounted under the equity method

53. For the purpose of starting the working capital of Alcano Corp. on Dec. 31, 2022, the following data are
submitted:

Cash on hand and in bank, net of bank overdraft of P50,000 P 550,000


Petty cash (unreplenished petty cash expenses, P4,000) 10,000
Notes receivable 750,000
Accounts receivable, net of accounts with credit balances of P100,000 1,100,000
Merchandise inventory, excluding unrecorded purchase of P200,000 on
account in transit on Dec. 31, 2022 and shipped FOB shipping point on
Dec. 31, 2022 2,000,000
Prepaid expenses 90,000
Accounts payable, net of accounts with debit balances of P150,000 3,000,000
Note payable, with annual installment of P500,000 payable every May 31 2,000,000
Accrued expenses 300,000

The total current liabilities of Alcano Corp. on Dec. 31, 2022 should be
a. P5,800,000 c. P4,100,000
b. P3,950,000 d. P4,300,000

54. Which statement is true according to PAS10 Events after the reporting period?
a. A decline in the market value of investments would normally be classified as an adjusting event.
b. The settlement of a long-running court case would normally be classified as a non-adjusting event.
c. Notes to the financial statements should give details of all material adjusting events included in those
financial statements.
d. Notes to the financial statements should give details of material non-adjusting events which could influence
the economic decisions of users.

55. Items of other comprehensive income least likely arise from


a. Defined benefit plans
b. Non-financial assets
c. Leases
d. Insurance contracts

56. Which of the following is currently defined in the statement of profit or loss?
a. Operating Income
b. Earnings Before Interest and Tax (EBIT)

Page 13 of 18
c. Profit or Loss
d. None of these

57. The B Corporation presented the following multiple-step income statement and statement of retained
earnings for the year ended Dec. 31, 2022, as developed by its bookkeeper who has completed 12 units of
accounting:

B Corporation
Revenue Statement
December 31, 2022

Net Sales P390,000


Less: Dividends declared P3.50 per ordinary share 15,000
Revenues P375,000
Less: Selling expenses 41,600
Gross profit P333,400
Less: Operating expenses:
Interest expense P 8,200
Cost of goods sold 227,400
Provision for income tax 23,920
Net operating income P 73,880
Add: Dividend income 3,600
Less: General and administrative expense 48,600
Net profit P 28,880

B Corporation
Retained Earnings Statement
December 31, 2022

Beginning retained earnings P116,000


Add: net profit 28,880
Adjusted retained earnings P144,880
Less: Loss on sale of land 8,000
Ending retained earnings P136,880

The correct net profit is


a. P43,880 c. P34,000
b. P39,000 d. P35,880

58. Dessa Corp. is preparing its statement of cash flows for the year ended Dec. 31, 2022 and has provided the
following information:

Profit before income tax P880,000


Depreciation on property, plant and equipment 250,000
Provision for impairment losses 150,000
Unrealized foreign exchange gains 60,000
Fair value adjustment gain on FA at FVTPL 130,000
Fair value adjustment loss on FA at FVTOCI 65,000
Fair value adjustment loss on investment property 190,000
Share of profit of associate 220,000
Gain on sale of FA at AC 85,000
Loss on sale of equipment 70,000
Gain on debt extinguishment 125,000
Gain on distribution of non-cash assets to owners 40,000
Interest expense 150,000
Interest payable, beginning of the year 100,000
Interest payable, end of the year 50,000
Page 14 of 18
Interest income 80,000
Interest receivable, beginning of the year 30,000
Interest receivable, end of the year 20,000
Income taxes paid 260,000
Accounts receivable, beginning of the year 500,000
Accounts receivable, end of the year 850,000
Inventory, beginning of the year 500,000
Inventory, end of the year 400,000
Accounts payable, beginning of the year 200,000
Accounts payable, end of the year 500,000

The change in FA at FVTPL represents the fair value adjustment.

Compute for the net cash provided by operating activities.


a. P730,000 c. P795,000
b. P770,000 d. P880,000

59. A diversified entity, which is listed in the Philippine Stock Exchange, is in the process of determining its
reportable segments for purposes of complying with PFRS 8. The performance of the segments reflects the
impact of the corona virus pandemic. The following data pertain to the entity’s eight operating segments:

Segment Profit Loss


1 P14,000,000 -
2 9,000,000 -
3 P16,000,000
4 12,500,000
5 6,000,000 -
6 - 3,200,000
7 2,000,000 -
8 - 1,500,000
P31,000,000 P33,200,000

In the entity’s segment information, which is(are) reportable segment(s)?


a. Segments 1, 2, 3, 4, 5 & 6
b. Segments 1, 2, 3, 4 & 5
c. Segments 1, 2, & 5
d. Segments 3 & 4

60. The following are related parties of the reporting entity, except
a. Daughter of a person who has significant influence over the reporting entity.
b. Major customer of the reporting entity.
c. An entity providing key management personnel services to the reporting entity.
d. Post‐employment benefit plan for the benefit of employees the reporting entity.

61. Joy Corp. operates in hyperinflationary economy. Its statement of financial position at Dec. 31, 2022 reported
the following items:

Cash and cash equivalents P 200,000


Trade and other receivables 350,000
Inventories 500,000 450/400
Prepayments 50,000 450/180
Debt investments at amortized cost 200,000
Property, plant and equipment at cost 1,000,000 450/150
Accumulated depreciation ( 300,000) 450/150
Investment property at fair value 600,000 450/160
Trade and other payables 250,000
Short-term borrowings 180,000
Page 15 of 18
Note payable to bank 800,000
Share capital 400,000 450/100
Retained earnings 970,000

Data about the general price indices:

Dec. 31, 2022 – 450


July 1, 2022 – 400
Dec. 31, 2021 – 180
Dec. 31, 2020 – 160
Dec. 31, 2019 – 150
Dec. 31, 2018 – 100

Additional information:

• Inventories at Dec. 31, 2022 were acquired on July 1, 2022.


• The prepayments were made on Dec. 31, 2021.
• Property, plant and equipment were acquired on Dec. 31, 2019.
• Investment property was acquired on Dec. 31, 2020

What is the retained earnings balance on Dec. 31, 2022 after adjusting for hyperinflation?
a. P1,032,500 c. P1,385,714
b. P1,107,500 d. P1,907,500

62. Which statement is incorrect regarding interim financial reporting?


a. PAS 34 prescribes the minimum content of an interim financial report.
b. An entity may reverse impairment loss on goodwill recognized in the first quarter in subsequent quarters
of the same financial reporting period.
c. Seasonal revenues of retailers are recognized when they occur.
d. The preparation of interim financial reports generally will require a greater use of estimation methods
than annual financial reports.

63. Which statement is correct regarding PFRS for SMEs?


a. The FRSC expects to consider amendments to the PFRS for SMEs for approximately once every five
years.
b. An SME which has a subsidiary that is mandated to report under the full PFRSs may apply, at its option,
the full PFRSs.
c. Accounting for derivatives is one of the omitted topics since derivatives are not relevant to typical SMEs.
d. The revaluation model is not allowed in PFRS for SMEs.

Use the following information for the next two questions.

An SME acquired an investment in equity instrument for P500,000 on June 30, 2022. The direct acquisition costs
incurred were P25,000.

On Dec. 31, 2022 the fair value of the instrument was P600,000 and the transaction costs that would be incurred
on sale were estimated at P30,000.

64. If the fair value of the investment can be measured reliably without undue cost or effort, it should be reported
on the Dec. 31, 2022 statement of financial position at
a. P500,000 c. P570,000
b. P525,000 d. P600,000

65. If the fair value of the investment cannot be measured reliably without undue cost or effort and the best
estimate of the amount that the entity would receive for the sale of the investment is P600,000, the investment
should be reported on the Dec. 31, 2022 statement of financial position at
Page 16 of 18
a. P500,000 c. P570,000
b. P525,000 d. P600,000

66. Which of the following apply both to medium-sized and small entity?
a. There is no accounting for onerous contracts.
b. No line items specified in the statement of financial position.
c. Biological assets can be carried either at cost or at current market price, depending on the policy choice
made by the entity.
d. Disclosure of key management personnel compensation is required.

67. On June 30, 2020, Entity A (a small entity) acquired 30 per cent of the ordinary shares that carry voting rights
of Entity X for P200,000. In acquiring those shares the entity incurred transaction costs of P2,000.

The entity uses the cost method to account for its investments in associates.

In Jan. 2021 Entity X declared and paid a dividend of P40,000 out of profits earned in 2020. No further
dividends were paid in 2021 and 2022.

A published price quotation does not exist for Entity X. At Dec. 31, 2020, 2021 and 2022, in accordance with
Section 21 of the PFRS for SEs, management assessed the fair value of its investment in Entity X as
P204,000, P220,000 and P180,000 respectively. Costs to sell are estimated at P8,000 throughout.

In relation to this investment, how much is the net amount that Entity A should recognize in its profit or loss
for the year ended Dec. 31, 2020, 2021 and 2022 respectively:

a. P(6,000), P18,000, P(30,000)


b. P(6,000), P6,000, P(30,000)
c. P(4,000), P18,000, P(30,000)
d. P196,000, P202,000, P172,000

68. Which of the following regarding accrual versus cash-basis accounting is true?
a. The FRSC believes that the cash basis is appropriate for small and medium-sized entities, especially
those in the service industry.
b. The cash basis is less useful in predicting the timing and amounts of future cash flows of an entity.
c. Application of the cash basis results in an income statement reporting accrued revenues and expenses.
d. The cash basis requires a complete set of double-entry records.

Use the following information for the next two questions.

The trial balance of an entity does not balance.

Debit Credit
Cash P 5,912
Accounts Receivable 5,240
Supplies on Hand 2,967
Furniture and Equipment 6,100
Accounts Payable P 7,044
Share capital 8,000
Retained Earnings 2,000
Service Revenue 5,200
Office Expense 4,320 _______
P24,539 P22,244

Page 17 of 18
An examination of the ledger shows these errors.
• Cash received from a customer on account was recorded (both debit and credit) as P1,380 instead of
P1,830.
• The purchase on account of a computer costing P3,200 was recorded as a debit to Office Expense and a
credit to Accounts Payable.
• Services were performed on account for a client, P2,250, for which Accounts Receivable was debited P2,250
and Service Revenue was credited P225.
• A payment of P95 for telephone charges was entered as a debit to Office Expenses and a debit to Cash.
• The Service Revenue account was totaled at P5,200 instead of P5,280.

69. The corrected trial balance of the entity should show total debits of
a. P24,349 c. P24,729
b. P24,444 d. P27,549

70. The net income for the year is


a. P6,185 c. P6,815
b. P6,285 d. P6,850

End of Examination
(Pleases ignore the extra answer options in the answer sheet after number 70)

Thank you for participating in


Team PRTC Nationwide Online Open Final Pre-Board Examination for May 2022 LECPA!

Page 18 of 18
ADVANCED FINANCIAL ACCOUNTING AND REPORTING DE LEON/DE LEON/ALENTON
FINAL PRE-BOARD EXAMINATION April 26, 2022

Multiple Choice. Select the letter that corresponds to the best answer. This examination consists of
70 items only. (Please ignore the extra answer options in the answer sheet after number 70.) The exam
is good for three (3) hours. Good luck!

1. A partnership's income-sharing ratio


a. applies to partnership income after salaries and interest are deducted
b. applies to partnership income before salaries are deducted.
c. applies partnership income after salaries are deducted but before interest is deducted.
d. applies to partnership income before both salaries and interest are deducted.

2. When can the bonus method be applied?


a. When a partnership is formed
b. When a new partner is added to the partnership
c. When an existing partner retires from the partnership
d. The bonus method can be applied in all three of the above circumstances

3. In what manner do the remaining partners share in the bonus paid toa withdrawing partner?
a. In proportion to, their residual profit and loss ratios
b. Equally
c. In proportion to their capital account balances
d. The partner with the greatest capital account is assigned the bonus
The following condensed balance sheet is presented for the partnership of Art and Bea, who share profits and
losses in the ratio of 60:40, respectively:
Cash P 45,000 Accounts payable P 120,000
Other assets 625,000 Art, capital 348,000
Bea, loan 30,000 Bea, capital 232,000
Total P 700,000 Total P 700,000

The assets and liabilities are fairly valued on the balance sheet. Art and Bea decide to admit Ces as a new
partner with 20% interest.

4. What amount should Ces contribute in cash or other assets?


a. P110,000 c. P140,000
b. P116,000 d. P145,000

5. Instead of admitting a new partner, Art and Bea decide to liquidate the partnership. If other assets are sold
for P500,000, what amount of the available cash should be distributed to Art?
a. P255,000 c. P327,000
b. P273,000 d. P348,000

On December 31, 1998, the partners of MNP Partnership decided to liquidate their business. Immediately
before liquidation, the following condensed balance sheet was prepared:

Cash P 50,000 Liabilities P 375,000


Noncash assets 900,000 Nieva, loan 80,000
Perez, loan 25,000
Munoz, capital (50%) 312,500
Nieva, capital (30%) 107,500
____ Perez, capital (20%) 50,000
Total P 950,000 Total P 950,000

6. The noncash assets were sold for P400,000. Assuming Perez is the only solvent partner, what amount of
additional cash will be invested by Perez? (rounded to the nearest peso)
Page 1 of 14
a. P 37,143
b. 25,000
c. 5,250
d. 0

The partners of the R & E Partnership started liquidating their business on July 1, 2022, at which time the
partners were sharing profits and losses 40% to R and 60% to E. The balance sheet of the partnership
appeared as follows:

R & E Partnership
Balance Sheet – July 1, 2022

Assets Liabilities & Capital


Cash……………………. P 8,800 Accounts payable………… P32, 400
Receivable……………… 22,400 R, capital………………… P31, 000
Inventory…………...….. 39,400 R, drawing………… 5,400 25, 600
Equipment…. P65, 200 E, capital………………… .P33, 200
Accumulated E, drawing……………………. 200 33, 000
depreciation 30, 800 34, 400 E, loan…………………………………… 14, 000
Total…………………… P105, 000 Total……………………………… P105, 000

During the month of July, the partners collected P600 of the receivables with no loss. The partners also sold
during the month the entire inventory on which they realized a total of P32,400.

7. How much of the cash was paid to R’s capital on July 31, 2022?
a. P -0- c. P5, 400
b. 25, 600 d. 320

8. It refers to the implementation of a business plan to restructure or rehabilitate a corporation with the hopes
of increasing company value. In most cases, it involves changing the entity’s capital structure.
a. transformation c. reorganization
b. mutation d. translation

9. The total unsecured liabilities without priority can be computed as


a. Unsecured creditors without priority plus deficiency of assets pledged to partially secured creditors
b. Unsecured creditors without priority less estimated realizable value of assets pledged to partially secured
creditors
c. Sum of administrative expenses, unpaid employee salaries and benefits, and taxes and assessments.
d. Total liabilities less priority claims.

10. It is a financial report which shows information on the progress of the liquidation process of a corporation.
a. statement of affairs c. statement of realization and liquidation
b. statement of liquidating affairs d. statement of changes in net assets

The following was taken from the Statement of Affairs of Paradigm Company at August 30, 2022:

Assets pledged with fully secured creditors P 71,000


Assets pledged with partially-secured creditors 12,500
Free assets 11,000
Liabilities with priority 3,000
Fully - secured liabilities 69,000
Partially-secured liabilities 20,000
Liabilities without priority 18,000

11. The estimated deficiency to unsecured amounts is


a. P 15,500 c. P10,550
b. P 15,150 d. P10,050
Page 2 of 14
12. The estimated amount payable to partially-secured liabilities is
a. P14,450 c. P15,441
b. P 14,045 d. P14,540

13. The estimated amount payable to liabilities without priority is


a. P7,059 c. P9,750
b. P 5,790 d. P9,570

The following data were taken from the statement of realization and liquidation of Mendoza Corp. for the
quarter ended September 30, 2031

Liabilities to be liquidated 285,000


Supplementary charges 169,100
Liabilities not liquidated 210,000
Supplementary credits 192,500
Assets acquired 136,000
Liabilities liquidated 158,000
Assets to be realized 107,500
Assets realized 175,000
Liabilities assumed 83,000

The beginning capital balances of ordinary shares and retained earnings are P102,000 and P29,600,
respectively. A net income of P87,400 for the period.

14. How much is the beginning balance of cash?


a. P293,000
b. 209,100
c. 241,600
d. 309,100

15. Statement 1 (S1): The balance of the Allowance for Overvaluation of Inventories: Branch ledger account is
deducted from the balance of the Investment in Branch account in the separate balance sheet of the home
office.
Statement 2 (S2]: If the home office bills shipments of merchandise to the branch ai 25% above home office
cost and the adjusted balance of the Allowance for Overvaluation of Inventories: Branch ledger account is
P20,400, the amount of branch inventories at billed prices is P81,600.
a. S1 - True; S2 - True
b. S1 - True; S2 - False
c. S1 - False: S2 - True
d. S1 - False: S2 - False

Home office bills its branch for merchandise shipments at 30% above cost.
The following are some of the account balances on the books of home office and its branch as of December 31,
20X0:
Home Office Books Branch Books
Inventory, January 1 35,000 101,500
Shipments from Home Office 263,900
Purchases 1,575,000 350,000
Shipments to Branch 253,750
Branch Inventory Allowance 91,875
Sales 2,100,000 1,260,000
Operating Expenses 507,500 192,500

Per physical count, the ending inventory of the branch is P73,500 including goods from outside purchases
of P48,475; the ending inventory of the home office is P210,000.
Page 3 of 14
16. What is cost of goods available for sale of the branch?
a. P715,400
b. P781,375
c. P689,500
d. P638,750

17. What is the total ending inventory to be shown on the combined financial statements?
a. P118,475
b. P277,725
c. P328,475
d. P280,000
.
18. What is the combined net income for the year?
a. P957,950
b. P871,850
c. P891,975
d. P942,725

PAPSY CORPORATION purchases all the outstanding shares of MAMSHIE COMPANY on January 2, 2021 for
P385,000 cash. On this date the stockholders equity of MAMSHIE is as follows:

Share capital, P10 par P175,000


Paid-in capital in excess of par 87,500
Retained earnings 175,000

Any excess of the fair value of net assets over the fair value of the investment is attributable to MAMSHIE’s
building which is currently overstated in its books. All other net asset items of the acquired company are fairly
valued at the acquisition date. The building has an estimated life of 10 years from January 2, 2021 without
salvage value.

The condensed trial balances of the affiliated companies on December 31, 2021 appear as follows:

PAPSY MAMSHIE
Current assets P 420,000 P 302,750
Land 210,000 210,000
Building (net) 1,050,000 283,500
Investment in MAMSHIE 385,000 -
Current liabilities (708,750) (367,500)
Ordinary shares, P3 par (525,000) -
Share capital, P10 par - (175,000)
Paid-in capital in excess of par (315,000) ( 87,500)
Retained earnings, Jan. 2, 2021 (446,250) (175,000)
Sales (367,500) ( 70,000)
Cost of goods sold 210,000 61,250
Operating expenses 78,750 17,500
Dividends declared 8,750 -
Totals -- --

19. Compute the consolidated net income for 2021.


a. P75,520 c. P72,550
b. P70,525 d. P75,250

20. Compute the consolidated Retained Earnings at December 31, 2021.


a. P517,250 c. P515,270
b. P525,170 d. P512,750

Page 4 of 14
21. Working paper eliminations are entered in
a. Both the parent company’s and the subsidiary’s accounting records
b. The parent company’s accounting records only
c. Neither the parent company’s nor the subsidiary’s accounting records
d. The subsidiary’s accounting records only

22. What date should be used as the acquisition date for a business combination?
a. The date when the acquirer signs the contract to purchase the business
b. The date when the acquirer obtains control of the acquiree
c. The date when all the contingencies related to the transaction are resolved
d. The date when the acquirer purchased more than 20% of the stock of the acquiree

On January 1, 2021, GININTUANG PUSO CORPORATION acquired 80% of the outstanding shares of BAGAL
SULONG COMPANY for P743,750. At this date, the stockholders’ equity of BAGAL SULONG follows:

Ordinary shares, P5 par P 350,000


APIC 175,000
Retained earnings 175,000
P 700,000

The net assets of BAGAL SULONG on January 1, 2021 were fairly valued. GININTUANG PUSO assigned the full
fair value to the non-controlling interest at the date of acquisition in analyzing the fair value of its investment.

Selected information over the first two (2) years of affiliated operations follows:

• Intercompany merchandise sales are summarized as follows:


Purchaser’s
Date Transaction Sales Amount GPR Remaining Ending
Inventory
In 2021 Downstream P 61,250 30% P 15,750
Upstream 35,000 25% 6,125
In 2022 Downstream 56,000 30% 10,500
Upstream 52,500 25% 5,250

• Condensed trial balances of the two (2) companies on December 31, 2018 follow:
GININTUANG BAGAL SULONG
PUSO CORP. COMPANY
Current assets P 1,428,000 P 387,275
Investment in BAGAL SULONG 743,750 ---
Equipment, net 1,891,750 262,500
Buildings, net 1,592,500 332,500
Goodwill 105,000 ---
Liabilities (1,123,500) (186,025)
Common Stocks, P1 par ( 437,500) ---
Ordinary shares, P5 par (350,000)
APIC (2,187,500) (175,000)
Retained earnings, January 1, 2022 (1,933,750) (245,000)
Sales (1,540,000) (1,102,500)
Dividend income ( 42,000) ---
Cost of goods sold 1,232,000 882,000
Other expenses 227,500 141,750
Dividends declared 43,750 52,500
Totals P 0 P 0

23. Compute the consolidated net income for 2022.


a. P 195,125 c. P215,915
b. P 251,195 d. P161,043
Page 5 of 14
24. Compute the amount of the consolidated net income for 2018 attributable to the parent’s shareholders.
a. P 175,500.50 c. P195,030
b. P 145,249 d. P143,482.50

25. Compute the amount of consolidated net income attributable to the non-controlling interest
a. P 19,624.50 c. P 20,885
b. P 15,794 d. P 15,732.50

On January 1, 2020, West Corporation purchased 80% of the common stocks of Fast Company. Separate balance
sheets for the companies at acquisition date are as follows:
West Corp Fast Co. FMV
Cash P 12,000 P 119,000
Accounts receivable 72,000 13,000
Inventory 66,000 19,000 29,000
Plant assets, net 230,000 120,000 140,000 (remaining life – 5 years)
Investment in Fast 196,000 _________
Total assets P 615,000 P 271,000
Accounts payable P103,000 P 71,000
Capital stock 400,000 150,000
Retained earnings 112,000 50,000
Total equities P 615,000 P 271,000

26. Goodwill (income) to be recorded by West Corp.


a. P 45,000 c. P 0
b. (P 26,000) d. (P 34,000)

27. Total assets on the consolidated balance sheet


a. P 735,000 c. P 720,000
b. P 931,000 d. P 705,000

28. Non-controlling interest in net assets as of January 1, 2020


a. P 39,200 c. P 46,000
b. P 40,000 d. P 49,000

CORPORATION will present consolidated financial statements in its Philippine peso functional currency.
In this connection, selected items from the Thailand subsidiary’s financial statements translated / re-measured
in Philippine peso are as follows:
Historical Closing Average
Accounts receivable Pp 12,000 Pp 12,500 Pp12,250
Inventories 15,000 17,500 16,250
Furniture and equipment, net
22,500 23,000 22,750
Patents 75,000 78,000 76,500
Marketable securities, at
market 5,000 5,500 5,250
Prepaid rent 10,000 12,500 11,750
Revenues - 850,000 820,000
Cost of sales - 500,000 525,000
Various expenses 250,000 212,500
Depreciation expense 2,000 2,200 2,100

29. Compute the amount of total assets that must be reflected in the consolidated balance sheet.
a. P149,000 c. P140,500
b. P140,000 d. P194,500

Page 6 of 14
30. Compute the net income that must be reflected in the consolidated income statement.
a. P 84,000 c. P 80,500
b. P 80,400 d. P 85,000

31. The currency of the primary economic environment in which the entity operates is called
a. Functional currency
b. Presentation currency
c. Foreign currency
d. Local currency

32. Which of these considerations would not be relevant in determining the entity's functional currency?
a. The currency that influences the costs of the entity
b. The currency in which finance is generated
c. The currency in which receipts from operating activities are retained
d. The currency that is the most internationally acceptable for trading

The TAGALOG CORPORATION purchases merchandise from a foreign exporter on November 15, 2017 for FC
100,000, payable in that currency on January 15, 2018. TAGALOG predicts the Philippine peso might probably
weaken against the foreign currency over the 60-day period and entered into a hedge of the exposed foreign
currency amount against the risk of exchange losses. The hedging relationship was designed in a way that it
would be fully effective thru the entire hedge period, and selected a forward contract for that purpose.
The following are relevant spot and forward rates at selected dates.

11/15/17 12/31/17 01/15/18


Spot rates P0.4295 P0.4245 P0.4345
Forward rates P0.4325 P0.4300 P0.4345

33. How much should TAGALOG CORPORATION report as transaction gain or (loss) on its Accounts payable
(in FC) at December 31, 2017 (its current year-end)?
a. P500 c. P(300)
b. P250 d. P(250)

34. How much amount was Due from the Broker as at December 31, 2017?
a. P43,000 c. P42,450
b. P43,250 d. P43,450

35. How much transaction gain or (loss) should TAGALOG CORPORATION report when the Accounts
payable (in FC) was settled in 2018?
a. P500 c. P (500)
b. P1,000 d. P(1,000)

36. How much was the transaction gain or (loss) recognized on the forward contract at December 31, 2017?
a. P500 c. P(300)
b. P250 d. P(250)

37. How much was the over-all transaction gain or (loss) over the hedging relationship.
a. P500 c. P(300)
b. P250 d. P(250)

Page 7 of 14
The SAN AGUSTIN BUILDERS started work on three job sites during the current year. Any costs incurred are
expected to be recoverable. Data relating to the three jobs are given below:
Estimated Collection from
Contract Actual Cost Cost to Billings on Customers
Site Price Complete Contract
Cebu P 500,000 P 375,000 - P 500,000 P 500,000
Bohol 600,000 254,167 P 381,250 180,000 100,000
Davao 250,000 100,000 100,000 150,000 100,000

38. Calculate the net amount to be reported on the balance sheet for the above projects under the
percentage of completion method:
a. Contract Liability P (11,250)
b. Contract Liability P (13,750)
c. Contract Asset P 13,750
d. Contract Asset P 11,250

39. Calculate the net amount to be reported on the balance sheet for the above projects under the zero
profit method.
a. Contract Liability P (11,250)
b. Contract Liability P (13,750)
c. Contract Asset P 13,750
d. Contract Asset P 11,250

CONCRETE CONSTRUCTION COMPANY began construction work in 2021 for a project with a contract price of
P8,000,000. CONCRETE uses the cost-to-cost percentage of completion method. The financial for 2021 relating
to the contract shows the following:

Accounts receivable P 500,000


Construction in Progress 1,600,000
Progress billings to date 1,500,000
Gross profit earned in 2021 200,000

40. Calculate the following items for the year 2021.


Cash Collections Cost incurred to date
a. P 1,000,000 P 1,400,000
b. P 7,500,000 P 1,400,000
c. P 1,000,000 P 1,600,000
a. P 1,400,000 P 1,600,000

41. The percentage of completion of a construction contract is based on all of the following, except
a. The proportion that contract costs incurred for work performed to date bear to the estimated total
contract costs.
b. Survey of work performed.
c. Completion of a physical proportion of the contract work.
d. Progress payments and advances received from customers.

42. Financial statements of nonprofit organization (NPO) includes all of the following, EXCEPT
a. Statement of changes in equity
b. Statement of financial position
c. Statement of activities
d. Statement of cash flows

43. Net assets that are restricted by the governing board of a non-government, not-for-profit organization are
reported part of:
a. Unrestricted net assets
b. Any of these, depending on the terms
c. Permanently restricted net asset
Page 8 of 14
d. Temporarily restricted net assets

44. The Commission on Audit (COA) is responsible for


a. the formulation and implementation of the national budget with the goal of attaining the nation’s socio-
economic objectives.
b. receiving and keeping national funds and managing and controlling the disbursements thereof.
c. directly implementing the projects of the government.
d. promulgating accounting and auditing rules and regulations.

45. Entity A, a government entity, receives authorization to disburse funds not to exceed ₱1B in a specified
period. This event can be described as
a. Notice of Cash Allocation
b. Allotment
c. Appropriation
d. Adontknowcation

46. Entity A, a government entity, receives notice that out of its ₱10B approved budget for the year, Entity A can
incur obligations up to ₱4B in the first quarter. This event can be described as
a. Notice of Cash Allocation
b. Allotment
c. Appropriation
d. Amnotsurecation

47. Entity A, a government entity, receives notice that for the current year, the maximum amount it can spend
on maintenance and other operating expenses is ₱10B. This event can be described as
a. Notice of Cash Allocation
b. Allotment
c. Appropriation
d. Budgetication

BIGAY HILIG CORPORATION uses process costing in its operations and adopts the weighted average method in
costing its production. The Finishing department is the second and last stage before the product is transferred
to finished goods storage. Materials are added following inspection, which occurs at the end of the process.
Normal spoilage is expected to be 2% of good output (those that have passed quality control test).

The following data are available in relation to the production activities for the Finishing department for March,
2021.

Units Pesos
In- Process, March 1 (75% converted) 10,000
Cost from the preceding department P 38,000
Cost from this department
Materials 21,200
Labor and overhead 81,300
Received from the preceding department during March 40,000 140,000
Completed and transferred to finished goods 38,000
In-Process, March 31 (50% converted) 11 ,000
Lost units 1,000
Cost added in this department this month
Materials P 70,000
Labor and overhead 292,500

Page 9 of 14
Use the following analysis of actual units and equivalent units (EUP) for materials and conversion (determined
from the above data)

Equivalent Units of Production


Actual Units Materials Conversion

Completed and transferred


38,000 38,000 38,000
In-Process, March 31 11,000 --- 5,500
Total spoilage 1,000 --- 1,000
Total 50,000 38,000 44,500

48. Compute the total cost of completed units transferred to Finished goods.
a. P554,670 c. P545,760
b. P554,770 d. P557,640

49. Compute the cost of units recognized as actual overhead and charged to Factory Overhead Control.
a. P 11,960 c. P 11,690
b. P 2,870 d. P 8,720

50. Compute the cost of units still in-process at March 31, 2021.
a. P85,360 c. P 83,560
b. P86,530 d. P 83,650

51. In computing the current period’s manufacturing cost per equivalent unit, the FIFO method of process
costing considers current period costs
a. only
b. plus cost of beginning WIP
c. less cost of beginning WIP
d. plus cost of ending WIP

FOREVERMORE INDUSTRIES manufactures products X, A, and K from a joint production process at a cost of
P58,800. Additional information for the current period follows:

Units Sales Value After Estimated Costs Units


Products Produced Further to Complete To Sell Sold
Processing
X 800 P 40,000 P 5,000 P 4,000 700
A 600 34,200 4,000 4,200 382
K 400 32, 000 3,000 2,000 350
1,800 P 106,000 P12,000 P10,200 1,600

52. Compute the amount of Joint cost allocated to Product A under the NRV /market value method.
a. P18,900 c. P19,800
b. P21,700 d. P 18,200

53. Compute the unit cost for Product A under the NRV method of joint cost allocation.
a. P33.38 c. P 54.75
b. P37.00 d. P 38,33

DATING GAWI CORPORATION has two (2) service departments, namely Service A and Service B and also two
(2) production departments, namely Production I and Production II. The budgeted costs of the service
departments are allocated to the production departments on the basis of the number of employees. Shown
below are the number of employees and the budgeted amounts for each department.

Department # of Empl. Budgeted Costs


Service A 10 P 40,000

Page 10 of 14
Service B 25 60,000
Production I 265 480,000
Production II 250 520,000
Totals 550 P 1,100,000

The company is to use the step method in the allocation of service department costs to the producing
departments, to begin with Service A’s cost.

54. Compute the amount of cost allocated to Service B from the allocation of Service A’s cost.
a. P0 c. P 1,852
b. P1,143 d. P 1,413

55. Compute the amount of cost allocated to Service A from the allocation of Service B’s cost.
a. P0 c. P1,852
b. P1,143 d. P1,582

56. Compute the total amount of service department costs allocated to Production II.
a. P48,543 c. P51,457
b. P48,345 d. P51,475

LAS VEGAS MANUFACTURING COMPANY uses activity-based costing and identifies the following overhead
costs and cost drivers for the coming year 2020:

Overhead activity Activity cost driver Budgeted overhead Budgeted activity level
Machine set-up No. of set-ups P20,000 200 set ups
Inspection No. of inspection 130,000 6,500 inspections
Materials handling No. of material moves 80,000 8,000 moves
Engineering No. of engineering hours 50,000 1,000 hours

The following information is determined on two jobs that are expected to be completed in the ensuing year.

Job # 901 Job # 902


Direct materials P 5,000 P 12,000
Direct labor 2,000 2,000
Units to be produced 100 50
Number of set-ups 8 10
Number of inspections 22 15
Number of material moves 30 40
Number of engineering hours 25 50

57. Using ABC methods in assigning overhead into the cost of production, determine the total overhead cost
of Job 901
a. P2,790 c. P 2,640
b. P2,790 d. P 2,640

58. Activity-based costing (ABC) first assigns costs to:


a. Departments
b. Products
c. Activities
d. Overhead

During 2021, for Job #444, BILIS GAWA MANUFACTURERS incurred the following costs for a rush production
order of 400 units of Product Y for customer BOBBY TAN II:
Direct materials (P330 per unit) P 132,000
Direct labor (P400 per unit) 160,000
Factory overhead (applied at 150% of direct labor cost) 240,000
Total P 532,000
Page 11 of 14
Upon final inspection it was discovered that 25 units were defective and needed to be reworked for an estimated
total cost of P8,218. Another 15 units were spoiled and could no longer be reworked economically to the desired
specifications. The estimated fair value of the spoiled units was P15,960.

The production inefficiencies are deemed to have been a result of changing specifications by the customer.
However, the customer was willing to accept the remaining good units.

The company WILL REWORK the defective units and the customer’s order will be credited at 80% of the fair value
of the spoiled units.

59. Compute the unit cost of Job #444 under Option A above.
a. P 1,411 c. P 1,359
b. P 1,330 d. P 1,370

60. Now assume the company was the one at fault. The defective units were reworked and the spoiled units
were sold for P12,000. Compute the unit cost of Job #444.
a. P 1,411 c. P1,359
b. P 1,332 d. P1,330

61. DEF is the consignee for 1,000 units of product X for ABC Company. ABC should recognize the revenue from
these 1,000 units when
a. The agreement between DEF and ABC is signed.
b. ABC ships the goods to DEF.
c. DEF receives the goods from ABC.
d. DEF sells the goods and informs ABC of the sale.

A and B formed a joint operation. The following were the transactions during the year:

A B
Total purchases 400 320
Total sales 960 720
Expenses paid 800
Other income 40

The joint operation was completed at the end of the year. Each joint operator is entitled to a 10% commission
on its purchases and a 20% commission on its sales. Any remaining profit or loss is divided equally.

62. How much is the profit (loss) of the joint operation?


a. 200,000
b. (200,000)
c. 180,000
d. (180,000)

63. On the cash settlement between the joint operators,


a. A pays B ₱368
b. B pays A ₱368
c. A pays B ₱428
d. B pays A ₱428

64. Mr. Pyromaniac obtained two fire insurances for his house. During the year, Mr. Pyromaniac’s house was
burned. The legal principle that prohibits Mr. Pyromaniac from relaxing and just watch the house burn is
a. Principle of proximate cause.
b. Principle of contribution.
c. Principle of loss minimization .
d. Principle of indemnity.

Page 12 of 14
65. SKEPTIC DOUBTFUL Co. obtained a fidelity bond for its cashier. During the year, the cashier embezzled funds
of SKEPTIC. The legal principle that prohibits SKEPTIC from claiming compensation directly from the cashier
is
a. Principle of subrogation.
b. Principle of contribution.
c. Principle of loss minimization.
d. principle of indemnity.

Page 13 of 14
On August 1, 2018, Aircon, Inc. consigned 10 2-horsepower outmoded air-conditioning units to Argy Trading, a
new consignee, at a cost of P12,000 each. Aircon also paid P4,000 freight for the shipment of the consignment
to Argy. The selling prices are: for instant cash , P16,000 each; and for a 3-month account, P17,400, with a
minimum down payment of P5,800. The account sales are to be made available only to AAA credit=type
customers. Commission to the consignee is 8% of collection. The consignee paid delivery expenses on the sold
units for P2,400.

On August 31, Argy Trading reported sales of 8 units, 5 units for cash and 3 units on account, each credit
customer paying the P5,800 minimum cash down payment.

66. How much cash was remitted by Argy Trading to Aircon, Inc on August 31, 2018?
a. P69,808 c. P122,008
b. P87,208 d. P128,200

67. How much is the net profit recognized by Aircon, Inc. from the consignment sales of Argy Trading in
August, 2018?
a. P19,208 c. P29,080
b. P29,108 d. P 22,808

68. How much is the cost of the consigned inventory in the possession of the consignee?
a. P 24,800 c. P28,400
b. P20,480 d. P28,040

69. On December 1, 20x1, CANOROUS Co. granted a 5-year franchise right to MELODIOUS, Inc. for an initial
franchise fee of ₱400,000. The non-refundable initial franchise fee was collected in full upon signing of the
contract. As of December 31, 20x1, CANOROUS has no remaining obligation or intent to refund any of the
cash received, all of the services pertaining to pre-opening activities to set-up the contract have been
performed and there are no other material conditions or obligations required of CANOROUS under the
franchise agreement.
If the promise to grant the franchise right is distinct and that the performance obligation is satisfied at a
point in time, how much revenue shall CANOROUS recognize in December 20x1?
a. 400,000 c. 0
b. 80,000 d. None of these

70. If the promise to grant the franchise right is distinct and that the grant of franchise provides the customer
the right to access the entity’s intellectual property, how much revenue shall CANOROUS recognize in
December 20x1?
a. 400,000 c. 0
b. 80,000 d. 6,667

End of Examination
(Pleases ignore the extra answer options in the answer sheet after number 70)

Thank you for participating in


Team PRTC Nationwide Online Open Final Pre-Board Examination for May 2022 LECPA!

Page 14 of 14

You might also like